Download as pdf or txt
Download as pdf or txt
You are on page 1of 152

PATHOLOGY

MRCS Part B Notes by Mo

Infective Endocarditis (IE) ......................................................................................................................................................... 2


Aortic Stenosis ........................................................................................................................................................................... 6
Temporal Arteritis (Giant Cell Arteritis) (GCA) / Osteoporosis ............................................................................................... 8
Gangrene + Mesothelioma ..................................................................................................................................................... 12
Lung Cancer ............................................................................................................................................................................. 14
Tuberculosis (TB) ..................................................................................................................................................................... 16
Breast Cancer .......................................................................................................................................................................... 18
MEN 1 Syndrome .................................................................................................................................................................... 22
MEN 2 Syndrome .................................................................................................................................................................... 24
Parotid Tumors ........................................................................................................................................................................ 26
Nasopharyngeal Carcinoma.................................................................................................................................................... 32
Carcinoid Tumor ...................................................................................................................................................................... 34
Inflammatory Bowel Disease (IBD) ......................................................................................................................................... 35
Familial Adenomatous Polyposis (FAP) .................................................................................................................................. 41
GB Cancer + Pseudomembranous Colitis .............................................................................................................................. 43
Diverticulitis + Endometriosis ................................................................................................................................................. 45
Peptic Ulcer Disease + Hyperparathyroidism ........................................................................................................................ 49
Esophageal Carcinoma............................................................................................................................................................ 51
Gastric Carcinoma ................................................................................................................................................................... 53
Cancer Colon + Hemorrhoids + Myocardial Infarction ......................................................................................................... 55
Malignant Melanoma ............................................................................................................................................................. 59
Malignant Melanoma Metastasis........................................................................................................................................... 61
Basal Cell Carcinoma (BCC) ..................................................................................................................................................... 63
Sickle Cell Disease + Brain Tumor........................................................................................................................................... 65
Polycystic Kidney Disease (APKD) ........................................................................................................................................... 67
Testicular Teratoma ................................................................................................................................................................ 69
Prostate Cancer ....................................................................................................................................................................... 71
Abscess .................................................................................................................................................................................... 73
Osteomyelitis........................................................................................................................................................................... 75
Pathological Fractures ............................................................................................................................................................ 79
Polytrauma + Transfusion ....................................................................................................................................................... 81
Hepatitis C (HCV) ..................................................................................................................................................................... 83
Acute Pancreatitis ................................................................................................................................................................... 88
Summary of Definitions .......................................................................................................................................................... 90

Renal Cell carcinoma (Pathology)

Squamous Cell Carcinoma (SCC) On Arm

GIST tumor

Bleeding Pseudoaneurysm/ HIV

1
PATHOLOGY MO’s MRCS B NOTES (Previously called Reda’s Notes) 1
THE CONTENTS OF THIS SHEET:
❖ Mr Salah’s notes [MRCS Part B Notes by Mo].
❖ Mr Salah’s Mocks [JULY 2023].
❖ Mr Salah’s knowledge lectures.

IMPORTANT NOTES REGARDING THE


MOCKS:
• Mr Salah’s Mocks are written at the end of each note.
• Read the mocks carefully [more important than the notes].
• The stations in the exam will resemble the mocks.
• The mocks contain new questions and the most precise
answers.
• Try to repeat the mocks as much as you can.

📌OTHER IMPORTANT MATERIALS📌


➢ Other MRCS Part B materials
https://t.me/AwaserMRCSB
➢ MRCS Part A materials
https://t.me/AwaserMRCS
➢ OET materials
https://t.me/OETAwasir
Infective Endocarditis (IE)

Definition?
Inflammation of the endocardial surfaces of the heart including heart valves which is caused by certain microorganisms.

Types of endocarditis?
• Infective endocarditis: here microbes colonize the heart valves and form friable vegetations. The 2 types of IE
are acute and subacute. Diagnosis via Duke’s criteria
• Non-bacterial thrombotic endocarditis aka marantic endocarditis: this variant characteristically occurs in the
settings of cancers e.g. adenocarcinomas
• Libman sacks endocarditis: occurs in the settings of cancers e.g. adenocarcinoma

Major forms of vegetative endocarditis. The acute rheumatic fever phase of rheumatic heart disease (RHD) is marked by the appearance of
small, warty, inflammatory vegetations along the lines of valve closure; as the inflammation resolves, substantial scarring can result. Infective
endocarditis (IE) is characterized by large, irregular, often destructive masses that can extend from valve leaflets onto adjacent structures (e.g.,
chordae or myocardium). Nonbacterial thrombotic endocarditis (NBTE) typically manifests with small- to medium-sized, bland, nondestructive
vegetations at the line of valve closure. Libman-Sacks endocarditis (LSE) is characterized by small- to medium-sized inflammatory vegetations
that can be attached on either side of the valve leaflets; these heal with scarring.

Why rheumatic heart and valve replacement patients are more susceptible to IE?
Blood usually flows smoothly over valves, when these valves are damaged (as in RH) or in valve replacement, there will
be an increased chance for bacterial colonization on damaged tissues.

Pathophysiology of RHD?
• Acute rheumatic fever results from host immune responses to group A streptococcal antigens that cross-react
with host proteins. In particular, antibodies and CD4+ T cells directed against streptococcal M proteins can also
in some cases recognize cardiac self-antigens. Antibody binding can activate complement, as well as recruit Fc-
receptor bearing cells (neutrophils and macrophages); cytokine production by the stimulated T cells leads to
macrophage activation (e.g., within Aschoff bodies). Damage to heart tissue may thus be caused by a
combination of antibody- and T cell–mediated reactions
• Recurrent inflammation, progressive fibrosis, narrowing and stiffening of the valve leaflets with commissural
fusion, retraction of the leaflet edges, valve thickening, calcification leading to stenosis.

CD4+ TH1 cells (and sometimes CD8+ T cells, not shown) respond to tissue antigens by secreting cytokines that stimulate inflammation
and activate phagocytes, leading to tissue injury. CD4+ TH17 cells contribute to inflammation by recruiting neutrophils (and, to a lesser
extent, monocytes).

2
2 MO’s MRCS B NOTES (Previously Reda’s called Notes) PATHOLOGY
Gross findings?
Acute phase: Valvular vegetations (verrucae) along the lines of closure, having little effect on cardiac function
Chronic phase: Commissural fibrosis, valve thickening, and calcification + shortened and fused chordae tendinea (fish
mouth shape)

Microscopic findings? (Extra cellular matrix)


Aschoff bodies, a form of granulomatous inflammation which consists of a central zone of degenerating ECM infiltrated
by lymphocytes, plasma cells and Anitschkow cells (activated macrophages also termed as caterpillar cells due to wavy
nuclear outlines), found in all 3 layers of the heart – pericardium, myocardium or endocardium

What to see macroscopically?


• Aschoff nodules
• Fibrinoid necrosis

Rheumatic heart disease. (A) Acute rheumatic mitral valvulitis superimposed on chronic rheumatic heart disease. Small vegetations
(verrucae) are visible along the line of closure of the mitral valve leaflet (arrows). Previous episodes of rheumatic valvulitis have
caused fibrous thickening and fusion of the chordae tendineae. (B) Microscopic appearance of an Aschoff body in acute rheumatic
carditis; there is central necrosis associated with a circumscribed collection of mononuclear inflammatory cells, including some
activated macrophages with prominent nucleoli and central wavy (caterpillar) chromatin (arrows). (C and D) Mitral stenosis with
diffuse fibrous thickening and distortion of the valve leaflets, commissural fusion (arrows), and thickening and shortening of the
chordae tendineae. There is marked left atrial dilation as seen from above the valve (C). (D) Anterior leaflet of an opened rheumatic
mitral valve; note the neovascularization (arrow). (E) Surgically removed specimen of rheumatic aortic stenosis, demonstrating
thickening and distortion of the cusps with commissural fusion

Investigation to identify vegetations?


2D echo

What to look for in 2D echo?

3) Annular dilatation
!
"
#
$
%
&
'
(
)
*
+
,
'
-
(
+
,
&
,
&
(
.
%
,
%
-
-
/
0
1
*
%
)
&
(
.
%
2
,
+
3
.
0
&
,
+
4
,
/
0
4
(
&
5
0
(
6
%
(
1
(
)
,
%
&
'
$
6
*
'
6
(
&
,
&
(
.
%
"
1
)
V
a
l
v
u
l
a
r
r
e
g
u
r
g
i
t
a
t
i
o
n
:
A
r
e
g
u
r
g
i
t
a
n
t
j
e
t
>
1
c
m
i
n
l
e
n
g
t
h
a
n
d
p
e
a
k
v
e
l
o
c
i
t
y
>
2
.
5
m
/
s
!
"
#
$
%
&
'
$
(
)
*
+
,
'
%
-
.
$
/
0
,
%
-
(
%
(
1
,
2
&
%
1
'
3
+
$
/
4
5
1
6
7
$
2
1
2
8
9
:
;
<
<
"
/
=
$
>
3
6
$
>
<
,
?
1
'
1
(
@
/
A
,
>
3
'
$
.
!
"
#
$
%
&
'
(
)
*
)
%
+
,
(
-
.
%
+
/
&
0
1
-
0
&
*
5
)
I
n
c
r
e
a
s
e
d
e
c
h
o
g
e
n
i
c
i
t
y
o
f
s
u
b
v
a
l
v
u
l
a
r
a
p
p
a
r
a
t
u
s
!
"
#
$
%
&
'
(
%
)
&
(
*
$
+
+
,
-
&
.
/
3
PATHOLOGY MO’s MRCS B NOTES (Previously called Reda’s Notes) 3
Common organisms?
• Viridans Strep. or Staph. Streptococcus bovis
• Coagulase negative staph. Epidermidis
• Enterococci
• Hacek group of microorganisms (oropharyngeal commensals)
• (Haemophilus species, Aggregatibacter species, Cardiobacterium hominis, Eikenella corrodens, and Kingella
species.)

Diagnosis?
Dukes criteria
Selection Criteria (Diagnostic):
• 2 Major Criteria and 0 Minor Criteria
• 1 Major Criteria and 3 Minor Criteria
• 0 Major Criteria and 5 Minor Criteria

Major Criteria
Blood cultures positive for endocarditis
1.Typical microorganisms consistent with IE from 2 separate blood cultures, microorganisms consistent with IE
3.
from2.persistently positive blood cultures, single positive blood culture for Coxiella burnetii or phase I IgG
antibody titer >1:800.
Evidence of endocardial involvement
Echocardiogram positive for IE, abscess, new partial dehiscence of prosthetic valve, new valvular regurgitation.
Note: Worsening or changing of pre-existing murmur NOT sufficient.
Minor Criteria
Predisposition Fever
Heart condition or IV drug use Greater than or equal 38ᵒc
Vascular phenomena Immunologic phenomena
Major arterial emboli, septic pulmonary infarcts, Glomerulonephritis, Osler’s nodes, Roth’s spots, and
mycotic aneurysm, intracranial hemorrhage, rheumatoid factor.
conjunctival hemorrhages, and Janeway’s lesions.
Microbiological evidence Echocardiographic findings
Positive blood culture but does not meet a major consistent with endocarditis but do not meet a major
criterion as noted above or serological evidence of criterion as noted above
active infection with organism consistent with IE.

Causes and risk factors of IE?


• Acquired valvular heart disease with stenosis or • Cyanotic congenital heart defects
regurgitation • Colorectal cancer (Streptococcus bovis)
• Valve replacement • UTI (enterococci)
• Structural congenital heart disease, including • IVDA
surgically corrected, but excluding isolated ASD, • RHD
fully repaired VSD/PDA • HIV
• Previous IE • Malignancy
• HOCM (hypertrophic obstructive • DM
cardiomyopathy) • Alcohol
• Devices (Implantable cardioverter-defibrillators) • Tooth extractions.

Complications of IE?
Cardiac Non-cardiac
• AMI • GN
• Pericarditis (endocarditis-associated glomerulonephritis)
• Arrhythmia • AKI
• Valvular insufficiency • Stroke
• CCF (congestive cardiac failure) • Mesenteric/splenic abscess or infarct
• Sinus of Valsalva (aneurysm of the aortic sinus)
• Aneurysm
• Intra-cardiac abscess
• Arterial emboli

4
4 MO’s MRCS B NOTES (Previously Reda’s called Notes) PATHOLOGY
Blood invx?
- CRP
- ESR
Q: - Blood culture
Signs in hand? Signs and symptoms of IE FROM JANE
• Osler nodes: • Fever
Painful, raised, red lesions due to immune complex deposition • Roth's spots
• Janeway lesions: • Osler's nodes
Non-painful, nodular or macular red lesions due to septic emboli • Murmur
which deposit bacteria forming microabscesses • Janeway lesions
• Splinter hemorrhages: tiny blood clots under nails • Anemia
• Clubbing
• Nail hemorrhage (splinter hge)
• Emboli

Janeway lesion on palm of a 36 year old male with staphylococcus Osler nodes on index finger. Note also the tiny splinter
endocarditis haemorrhages on fingernail also seen in end

Treatment?
IV antibiotics depending on culture and sensitivity for 6 weeks (IV ceftriaxone and vancomycin)
Restrictions
• Valves do not have specific blood supply so antibiotics cannot reach
• Organisms lie inside the vegetations so antibiotics cannot reach
• Bacteria form a biofilm (glycocalyx covering) that shields them from antibiotics

If IE occurs in tricuspid valve in younger persons: → right sided heart failure

If no response to medical treatment?


Valve replacement or heart transplantation

Matching before heart transplantation?


HLA antigen HLA and ABO

If not matched?
Type 1 → graft rejection

How to prevent graft rejections?


Immunosuppressant therapy (TMS)
• Tacrolimus
• Mycophenolate
• Steroids

Side effects of long-term steroid?


• Opportunistic bacterial and viral infections such as EBV, CMV ---> leukemia, lymphoma
Cushinoid features: obesity, muscle weakness, hirsutism, striae
• Cardiovascular: fluid retention, hypertension

• Endocrine: DM
• Musculoskeletal: osteoporosis, AVN, proximal myopathy
This pt came after 6 years with high blast cells and lymphocytes, what do you suspect ?
- This pt may have lymphoproliferative disorder which may cause lymphoma or leukemia
Mechanism of action of immunosuppressants? How do immunosuppressants work?
The four classes of immunosuppressive drugs

5
PATHOLOGY MO’s MRCS B NOTES (Previously called Reda’s Notes) 5
Class Examples Action
Corticosteroids Prednisone, methyl prednisone Anti-inflammatory; kills T cells
Cytotoxic drugs Cyclophosphamide, azathioprine, methotrexate, Blocks cell division nonspecifically
leflunomide, mycophenolate mofetil, brequinar
sodium,
Immunophilins Cyclosporin Blocks T cell responses
Lymphocyte-depleting Antilymphocyte globulin, monoclonal antibodies Kills T cells nonspecifically, kills
therapies activated T cells

After valve replacement why on warfarin?


To prevent thromboembolism

Q: Mechanism of action of warfarin?


Vitamin K antagonist thus inhibiting clotting factors 2,7,9,10

How to monitor?
INR

Reversal?
• Vitamin k
• FFP
(If urgent surgery)• PCC (prothrombin complex concentrate)

Right sided vegetations = tricuspid valve IE = IV drug abuser

Prophylaxis against infective endocarditis


Antibiotic prophylaxis against infective endocarditis is not recommended routinely:
• for people undergoing dental procedures
• for people undergoing non-dental procedures at the following sites[2]:
o upper and lower gastrointestinal tract
o genitourinary tract; this includes urological, gynaecological and obstetric procedures, and childbirth
o upper and lower respiratory tract; this includes ear, nose and throat procedures and bronchoscopy.
Chlorhexidine mouthwash should not be offered as prophylaxis against infective endocarditis to people at risk of
infective endocarditis undergoing dental procedures.
Source: https://www.nice.org.uk/guidance/cg64
See also ASSCC page 79
Aortic Stenosis

Also see questions from the previous (IE) station…

Clinical picture?
See Pre-Operative Aortic stenosis station, ASSCC

Causes?
• Post-inflammatory scarring (rheumatic heart
disease)
• Senile calcific aortic stenosis
• Calcification of congenitally deformed valve

Cause of sudden death in AS?


• MI
• Aortic dissection (Ruptured)
Dystrophic calcification of the aortic valve. View looking down onto the
How stenosis occurs? unopened aortic valve in a heart with calcific aortic stenosis. It is
Lipid accumulation, inflammation, calcification → markedly narrowed (stenosis). The semilunar cusps are thickened and
valve thickening and stenosis fibrotic, and behind each cusp are irregular masses of piled-up
dystrophic calcification.
Q: what is aortic sclerosis?
- AV valve disease with insignificant gradient across the valve can progress to stenosis

Q: Why bicuspid valve has high risk of stenosis in a long term?


- Because the mechanical force that normally distributed to 3 valves is now distributed to 2 valves only.

Q: Why is bicuspid valve susceptible to IE?


- because in bicuspid valve there increased risk of stenosis and stasis of blood
6
6 MO’s MRCS B NOTES (Previously Reda’s called Notes) PATHOLOGY
Q: Pathophysiology? How does it leads to death?
• As the aortic valve progresses from sclerosis to stenosis, the left ventricle encounters chronic resistance to
systolic ejection (↑ afterload) → thickening of the left ventricular wall (hypertrophy)
• Effects of high left ventricular afterload include decreased left ventricular myocardial elasticity and coronary
blood flow and increased myocardial workload, oxygen consumption, and mortality.
Late manifestations of LVH include a smaller left ventricular chamber size, which decreases preload and worsens
systolic dysfunction. The result is insufficient stroke volume, cardiac output, and ejection fraction. Finally,
backward transmission of increased left ventricular pressure to the lungs may cause pulmonary venous
hypertension and reactive vasoconstriction of the pulmonary vasculature.

Aortic valve endocarditis, after a while weakness in
arm, why?
Thromboembolism which lead to cerebrovascular
stroke

Which coagulation system will not be affected by


warfarin initially?
Intrinsic pathway
Product of clotting pathway formed
from constituents of blood and
Define thrombus? remain in its same position
Thrombus is defined as solid material formed from the
constituents of blood in flowing blood
Q: What complication of non displaced thrombus? Infection & inflammation
Surgical options for valve replacement
• Mechanical valve – a long-lasting valve made
of durable materials
• Tissue valve (which may include human or
animal donor tissue)
• Ross Procedure – “Borrowing” healthy valve
and moving it into position of the damaged
Calcific valvular degeneration. (A) Calcific aortic stenosis of a previously normal
aortic valve valve (viewed from above the valve). Nodular masses of calcium are heaped up
Non-surgical: • TAVI/TAVR procedure – Transcatheter aortic within the sinuses of Valsalva (arrow). Note that the commissures are not fused,
valve replacement as in rheumatic aortic valve stenosis (B) Calcific aortic stenosis occurring on a
congenitally bicuspid valve. One cusp has a partial fusion at its center, called a
Alternative options include aortic valve balloon raphe (arrow). (C and D) Mitral calcification, with calcific nodules within the
valvuloplasty – the valve is widened using a balloon annulus (attachment margin) of the mitral leaflets (arrows). (C) Left atrial view.
(D) Section demonstrating the extension of calcification into the underlying
myocardium. Such involvement of adjacent structures near the interventricular
septum can impinge on the conduction system.

Q: If metallic valve replacement was done and the patient developed IE, why the valve should be removed?
• The valve will be a septic focus • It has no blood supply and It will form a biofilm and will be default to treat wit Abx

• The valve will be dehiscent


Vale vegetation can lead to thromboembolism
Q: If we found microscopic branching hyphae on a
removed metallic valve, what’s the cause?
This is fungal infection:
• Candida
• Aspergillus (Examiner waiting for this)
• Microsporum
• Trichophyton
• Epidermophyton
Mechanical valve Tissue valve
Mechanical vs Tissue valve?
Mechanical Tissue
Excellent durability (95% at 10 years) – low rate of reoperation Noiseless
Warfarin Do not need warfarin (At first 3 months only)
Easy to insert Insertion maybe more difficult
Thrombo-embolism 1-2 % / patient / year Low risk of thrombo-embolism (0 – 1%)
Bleeding risk 2% / pt / yr Low risk of bleeding (0 – 1%)
• Surgery of infected heart valve include?
Debridement of infected tissues, drainage of any abscesses, and repair of any fistulas that may develop in heart muscle

7
PATHOLOGY MO’s MRCS B NOTES (Previously called Reda’s Notes) 7
STATION 1:
Temporal Arteritis (Giant Cell Arteritis) (GCA) / Osteoporosis

What is GCA? SCENARIO:


Granulomatus Inflammatory disease of blood vessels (large and 60 y old female with headache (temporal
medium) of the head, mainly branches of ECA.
pain) and skull tenderness on mastication
and transient loss of vision.
Which part of vessel is affected most?
Tunica medium Biopsy from the temporal artery revealed
giant cell arteritis (granulomatous
Describe the pathological changes in microscopic picture? panarteritis with mononuclear cell infiltrates)
Morphology
• Involved arterial segments develop intimal thickening (with occasional thromboses) that reduces the luminal
diameter.
• Classic lesions exhibit medial granulomatous inflammation centered on the internal elastic lamina that produce
elastic lamina fragmentation.
• There is an infiltrate of T cells (CD4+ > CD8+) and macrophages.
• Although multinucleated giant cells are seen in approximately 75% of adequately biopsied specimens,
granulomas and giant cells can be rare or absent
• Inflammatory lesions are only focally distributed along the vessel and long segments of relatively normal artery
may be interposed

Temporal (giant cell) arteritis. (A) Hematoxylin-eosin-stained section of a temporal artery showing giant cells near the
fragmented internal elastic membrane (arrow), along with medial and adventitial inflammation. (B) Elastic tissue stain
demonstrating focal destruction of the internal elastic membrane (arrow) and medial attenuation and scarring.

One simple blood test to prove?


ESR (elevated)

Most confirmatory test?


Temporal artery biopsy (Before taking biopsy give high dose steroid)
Q: What is most important complication?
Blindness
Why blindness?
Ophthalmic artery involvement >> gives retinal artery (End artery)

8
8 MO’s MRCS B NOTES (Previously Reda’s called Notes) PATHOLOGY
Treatment?
Corticosteroids. Start prednisolone 60mg/d PO immediately or IV methylprednisolone if evolving visual loss or history of
amaurosis fugax. Typically a 2-year course.

One year later developed fracture NOF, why?


• AVN
• Osteoporosis

Why osteoporosis in this patient?


• Steroid therapy
• Post-menopausal

Pathological changes in osteoporosis?


• Histologically normal bone that is decreased in quantity.
• Postmenopausal osteoporosis the increase in osteoclast activity affects mainly bones or portions of bones that
have increased surface area, such as the cancellous compartment of vertebral bodies.
• The trabecular plates become perforated, thinned, and lose their interconnections, leading to progressive micro
fractures and eventual vertebral collapse

What is osteoporosis and what is its pathogenesis Metabolic bone disease of decrease bone mass of normally mineralized bone leading to fractures with
minimal trauma
Metabolic bone disease characterized by:
• Low bone mass
• Micro architectural deterioration of bone tissue
• Increase bone fragility
• Loss of bone matrix
Q: Three main mechanisms:
• Inadequate peak bone mass
• Excessive bone resorption
• Inadequate formation of new bone during bone turnover
Mechanism by which corticosteroids cause osteoporosis?
• Direct inhibition of osteoblast formation
• Direct stimulation of bone resorption
• Inhibition of GIT calcium absorption
• Stimulation of renal calcium losses
• Inhibition of sex steroids

Causes of osteoporosis?
o Primary
• Idiopathic
• Postmenopausal
• Senile
o Secondary
Endocrine Disorders Gastrointestinal Miscellaneous
• Addison disease • Hepatic insufficiency • Anemia
• Diabetes, type 1 • Malabsorption • Homocystinuria
• Hyperparathyroidism • Malnutrition • Immobilization
• Hyperthyroidism • Vitamin C, D deficiencies • Osteogenesis imperfecta
• Hypothyroidism Drugs • Pulmonary disease
• Pituitary tumors • Alcohol
• Neoplasia • Anticoagulants
• Carcinomatosis • Anticonvulsants
• Multiple myeloma • Chemotherapy
• Corticosteroids

9
PATHOLOGY MO’s MRCS B NOTES (Previously called Reda’s Notes) 9
Other causes of pathological fracture?
• Skeletal metastasis
• Paget's disease
• Multiple myeloma
• Rickets
• Osteomalacia
• Osteogenesis imperfecta
• Radiotherapy

What’s multiple myeloma?


It’s a plasma cell neoplasm commonly associated with lytic bone lesions, hypercalcemia, renal failure, and acquired
immune abnormalities. It produces large amounts of IgG 55% or IgA 25%. It is the most common primary bone tumor in
elderly

Diagnosis?
• Punched-out lytic skull lesions on x-ray
• M spike on protein electrophoresis
• Ig light chains in urine (Bence Jones proteins)
• CRAB:
o HyperCalcemia (corrected calcium > 2.75 mmol/l, > 11 mg/dL)
o Renal insufficiency attributable to myeloma
o Anemia (hemoglobin < 10 g/dl)
o Bone lesions (lytic lesions or osteoporosis with compression fractures)

What is Bence Jones protein?


Are monoclonal globulin proteins or immunoglobulin light chain found in the urine. The proteins are produced by
neoplastic plasma cells. Bence Jones proteins are present in 2/3 of multiple myeloma cases.

Concerns if going to surgery?


Addisonian crisis
See Steroids station, ASSCC

How to prevent?
• Increase the patient steroid dose prior to surgery
• Convert to IV hydrocortisone
Total hip arthroplasty

The patient went for THA, died in POD1 suddenly, cause?


Fat embolism

Cause of fat embolism? (In this pt)


• Long bone fracture (closed)
• Major Burns
• Acute pancreatitis
• DM
• Orthopedic surgery (intramedullary nailing, joint reconstruction)
• Decompression sickness
• CPBG (cardiopulmonary bypass graft)

How to manage?
Mainly supportive / prevention of complications like ARF, ARDS.
• Respiratory (O2/mechanical ventilation)
• Fluid and electrolytes balance
• General (DVT, sepsis, nutrition)
Specific unproven:
• Ethanol
• Dextran
• Heparin

10
10 MO’s MRCS B NOTES (Previously Reda’s called Notes) PATHOLOGY
STATION 7:
This page intentionally left blank

11
PATHOLOGY MO’s MRCS B NOTES (Previously called Reda’s Notes) 11
Gangrene + Mesothelioma

Define gangrene? SCENARIO:


Gangrene (or gangrenous necrosis) is a type of necrosis caused by a worker, smoker, toe gangrene
critically insufficient blood supply

Define necrosis?
Accidental and unregulated form of cell death resulting from damage to cell membranes and loss of ion homeostasis

Types of cell death?


• Necrosis
• Apoptosis
• Give me 4 deference between apoptosis and necrosis:
Feature Necrosis Apoptosis
Cell size Enlarged (swelling) Reduced (shrinkage)
Nucleus Karyopyknosis → karyorrhexis → karyolysis Fragmentation into nucleosome-size
fragments
Plasma membrane Disrupted Intact; altered structure, especially
orientation of lipids
Cellular contents Enzymatic digestion; may leak out of cell Intact; may be released in apoptotic bodies
Adjacent inflammation Frequent No
Physiologic or Invariably pathologic (culmination of Often physiologic, means of eliminating
pathologic role irreversible cell injury) unwanted cells; may be pathologic after
some forms of cell injury, especially DNA
damage

Not asked Pathogenesis of necrosis?


Severe/prolonged ischemia: severe swelling of mitochondria, calcium influx into mitochondria and into the cell with
rupture of lysosomes and plasma membrane. Death by necrosis due the release of cytochrome C from mitochondria

12
12 MO’s MRCS B NOTES (Previously Reda’s called Notes) PATHOLOGY
Post gangrene why there is redness?
- This is due to post-hypoxic vasodilation

Difference between dry and wet gangrene? Give me 4


Feature Dry Gangrene Wet Gangrene
Site Commonly limbs More common in bowel
Mechanism Arterial occlusion More commonly venous obstruction
Macroscopy Organ dry, shrunken and black Part moist, soft, swollen, rotten and dark
Putrefaction Limited due to very little blood supply Marked due to stuffing of organ with blood
Line of demarcation Present at the junction between healthy and No clear cut line of demarcation
gangrenous part
Bacteria Bacteria fail to survive Numerous present
Prognosis Generally better due to little septicemia Generally poor due to profound toxemia

What do you think is the cause of ischemia?


Atherosclerosis
Define atherosclerosis?
Pathological process of the vasculature in which an artery wall thickens as a result of accumulation of fatty
materials such as cholesterol
Risk factors?
• Smoking
• HTN
• DM
• Family history
• Increased LDL
Pathological cells of atheroma?
- macrophages and lymphocytes
Patient developed cough, one bedside test to do?
Sputum analysis

X-ray → pleural plaque


Q. Pleural plaques, the most common manifestation of asbestos exposure, are well-circumscribed plaques of dense collagen
that are often calcified
Significance? Bronchogenic CA
Increased malignancy risk of mesothelioma and lung adenocarcinoma

Give one classification of lung cancer? (Just mention the headings)


Non-small cell lung cancer Small cell lung carcinoma
These share common features of prognosis and Small cell lung carcinomas are comprised of cells with a
management. Paraneoplastic features and early disease neuro endocrine differentiation. The neuroendocrine
dissemination are less likely than with small cell lung hormones may be released from these cells with a wide
carcinoma. They comprise the following tumors: range of paraneoplastic associations. These tumors are
• Adenocarcinoma (40% cases) most common strongly associated with smoking and will typically arise in
lung cancer type encountered in never smokers. the larger airways. They disseminate early in the course
• Squamous cell carcinoma (25% cases) more slow of the disease and although they are usually
growing and are typically centrally located chemosensitive this seldom results in long lasting
• Large cell carcinoma (10% cases) remissions.
Best prognosis order? (SLAM)
Now the patient is presented with Mets, poorly differentiated, how to tell its epithelial origin? - Squamous
- Large cell
Immunohistochemistry or FISH technique - Adenocarenoma
- sMall cell
If the tumor was epidermal growth factor positive, what will be the chemotherapeutic agent?
Tyrosine kinase inhibitor (imatinib)

Types of necrosis
• Coagulative
• Liquefactive
• Caseous
• Fat
• Fibrinoid
• Gangrenous

13
PATHOLOGY MO’s MRCS B NOTES (Previously called Reda’s Notes) 13
STATION 8:
Lung Cancer See page 123 in ASSCC file

What other lung cancers do you know? SCENARIO: 70-year-old male smoker,
Small cell and non-small cell See previous station (Gangrene + Mesothelioma) COPD with small cell lung cancer.
Histology report saying carcinoid
What are the signs of aggressiveness on the report? invading pleura and lymph nodes.
• Invading pleura High mitotic rate
Focal necrosis
• Lymph nodes involved Increase pleomorphism

Pathogenesis of clubbing?
Various hypotheses have been proposed over the years to explain the pathophysiology of digital clubbing.
• Some research found significantly higher plasma growth hormone levels in patients with lung carcinoma and
clubbing than patients without clubbing.
• Megakaryocyte or platelet clusters, lodged in the peripheral vasculature of the digits, release platelet-derived
growth factor (PDGF) and lead to the increased vascularity, permeability, and connective tissue changes that are
the hallmark of clubbing

Pancoast paraneoplastic syndrome, what hormones?


ACTH causing Cushing syndrome

Now the patient presents with metastasis, poorly differentiated, how to tell its epithelial origin?
Immunohistochemistry or FISH technique

FISH technique?
Fluorescence in situ hybridization (FISH) is a kind of cytogenetic technique which uses fluorescent probes binding parts of
the chromosome that show a high degree of sequence complementarity. Fluorescence microscopy can be used to find
out where the fluorescent probe bound to the chromosome See Breast Cancer station

If the tumor was epidermal growth factor positive, what will be the chemotherapeutic agent?
Tyrosine kinase inhibitor (imatinib)

6 month later he came with back pain. Why?


Bone metastasis

What other tumor metastasis to bone?


BLT with kosher pickle - See Spine Anatomy file (Breast, Lung, Thyroid, Kidney, Prostate, Multiple Myeloma)

Define adenocarcinoma (Malignant tumor formed from glandular structures in epithelial tissue)
Adenocarcinoma is cancer that forms in mucus-secreting glands throughout the body. The disease may develop in many
different places, but it is most prevalent in the following cancer

Define emphysema
Emphysema is a lung condition that causes shortness of breath. In people with emphysema, the air sacs in the lungs
(alveoli) are damaged. Over time, the inner walls of the air sacs weaken and rupture — creating larger air spaces instead
of many small ones.

Paraneoplastic syndromes? Give examples?


Definition: Symptom complexes that occur in patients with cancer and that cannot be readily explained by local or distant
spread of the tumor or by the elaboration of hormones indigenous to the tissue of origin of the tumor are referred to as
paraneoplastic syndromes. Example: ACTH , hypercalcemia , SIADAH
• The most common paraneoplastic syndromes are hypercalcemia, Cushing syndrome, and nonbacterial
thrombotic endocarditis, and the neoplasms most often associated with these and other syndromes are lung
and breast cancers and hematologic malignancies.
• Paraneoplastic syndromes also may manifest as hypercoagulability, leading to venous thrombosis and
nonbacterial thrombotic endocarditis.
• Other manifestations are clubbing of the fingers and hypertrophic osteoarthropathy in patients with lung
carcinomas

14
14 MO’s MRCS B NOTES (Previously Reda’s called Notes) PATHOLOGY
Paraneoplastic Syndromes
Clinical syndrome Major Forms of Neoplasia Causal mechanism(s) / Agent(s)
Endocrinopathies
Cushing syndrome Small cell carcinoma of lung ACTH or ACTH or ACTH-like substance
ACTH-like substance
Pancreatic carcinoma
Neural tumors
Syndrome of inappropriate anti- Small cell carcinoma of lung; Anti-diuretic hormone or atrial
diuretic hormone secretion intracranial neoplasms natriuretic hormones
Hypercalcemia Squamous cell carcinoma of lung Parathyroid hormone–related
Breast carcinoma protein, TGF-α
Renal carcinoma
Adult T cell leukemia/lymphoma
Hypoglycemia Fibrosarcoma Insulin or insulin-like substance
Other mesenchymal sarcomas
Ovarian carcinoma
Polycythemia Renal carcinoma Erythropoietin
Cerebellar hemangioma
Hepatocellular carcinoma
Nerve and Muscle Syndrome
Myasthenia Bronchogenic carcinoma, thymoma Immunologic
Disorders of the central and Breast carcinoma, teratoma Immunologic
peripheral nervous systems
Dermatologic Disorders
Acanthosis nigricans Gastric carcinoma Immunologic; secretion of epidermal
Lung carcinoma growth factor
Uterine carcinoma
Dermatomyositis Bronchogenic and breast carcinoma Immunologic
Osseous, Articular, and Soft-Tissue Changes
Hypertrophic osteoarthropathy and Bronchogenic carcinoma Unknown
clubbing of the fingers
Vascular and Hematologic Changes
Venous thrombosis (Trousseau Pancreatic carcinoma Tumor products (mucins that activate
phenomenon) Bronchogenic carcinoma clotting)
Other cancers
Nonbacterial thrombotic endocarditis Advanced cancers Hypercoagulability
Anemia Thymoma Immunologic
Others
Nephrotic syndrome Various cancers Tumor antigens, immune complexes

Hormones in paraneoplastic syndrome?


Histologic type Paraneoplastic syndrome
Adenocarcinoma • Hypertrophic pulmonary osteoarthropathy
• Trousseau’s syndrome
Squamous cell carcinoma • Humoral hypercalcemia of malignancy (PTHrP)
• Pancoast syndrome
Small (oat) cell carcinoma • SIADH
• Lambert-Eaton myasthenic syndrome
• Cerebellar degeneration

15
PATHOLOGY MO’s MRCS B NOTES (Previously called Reda’s Notes) 15
Tuberculosis (TB)

Differential Diagnosis? SCENARIO: A young Indian lady came


• Hodgkin’s lymphoma (more in this age group and involves cervical back from a foreign travel with
lymph nodes more commonly) cervical lymphadenopathy (anterior
• TB triangle mass), loss of weight, night
• Infectious mononucleosis sweating
Which labs you will send her sputum to?
Microbiology + Cytology labs

What are the tests of TB? What are the 2 investigation for rapid detection of TB?
• Sputum examination (culture, Ziehl Neelsen stain) -- Recombinase Polymerase Amplification (RPA)
QuantiFERON (interferon gamma assays).
• Mantoux test - PCR
• PCR to differentiate mycobacteria tuberculosis from other species
• QuantiFERON (interferon gamma assays)
• FNAC of lymph node

How to label the sputum specimens?


Category-B UN3373

Where to put it?


In a biohazard bag (Red color)

Organism of TB?
• Mycobacterium tuberculosis
• Mycobacterium avium intracellulare (MAC) → disseminated infection in immunocompromised patients
• Mycobacterium bovis

What are the culture media for mycobacteria?


• Solid media: Lowenstein Jensen media, Middlebrook media
Liquid media: BACTEC/MGIT (mycobacteria growth indicator tube)

How long to culture?
1-8 weeks 6 to 12 week
Q: What are surgically relevant examples of granuloma?
What type of protein deposition? - Fungal infection
Amyloid AA - Crohn’s disease
- Foreign body.
- Aspergillosis.
Given the FNAC result: (necrotic tissue, histiocytes, giant cells), interpret? - Sarcoidosis
TB This is granuloma so most likely Tuberculosis - TB
- Leprosy
- RA
What are giant cells?
Multinucleated cells formed by fusion of monocytes/macrophages often forming granuloma
e.g. Langerhans’ giant cells, Reed sternberg cells
If the lady is diagnosed with TB, what is the next step?
Public health concern/ community concerns?
1- Notify the consultant in communicable disease control (CCDC) And notify health protection agency
2- Avoid working in the food factory Give anti-TB medication
3- Use mask during sneezing or coughing
4- DOTS (Directly Observed Treatment, Short-course) anti-TB therapy
5- Contact tracing: the identification and diagnosis of persons who may have come into contact with an infected
person for the last 21 days.

What is your advice to contacts?


Counselling, screening and treatment of other family members.

What is a granuloma?

.
O
r
g
a
n
i
z
e
d
c
o
l
l
e
c
t
i
o
n
o
f
m
o
d
i
f
i
e
d
m
a
c
r
o
p
h
a
g
e
s
r
e
s
e
m
b
l
e
s
e
p
i
t
h
e
l
i
a
l
c
e
l
l
s
a
n
d
s
u
r
r
o
u
n
d
e
d
b
y
r
i
m
o
f
l
y
m
p
h
o
c
y
t
e
s
w
i
t
h
m
u
l
t
i
n
u
c
l
e
a
t
e
d
g
i
a
n
t
c
e
l
l
s
16
16 MO’s MRCS B NOTES (Previously Reda’s called Notes) PATHOLOGY
Other Causes of granuloma?
• Leprosy
• Schistosomiasis
• Sarcoidosis
• Crohn’s
• Rheumatoid arthritis

The sequence of events in primary pulmonary tuberculosis, commencing with inhalation of virulent Mycobacterium tuberculosis
organisms and culminating with the development of cell-mediated immunity to the organism. A, Events occurring during early
infection, before activation of T-cell–mediated immunity. B, The initiation and consequences of T-cell–mediated immunity. The
development of resistance to the organism is accompanied by the appearance of a positive tuberculin test.

STATION 9:

17
PATHOLOGY MO’s MRCS B NOTES (Previously called Reda’s Notes) 17
Breast Cancer

[mammogram+ pathology report]

What can you recognize in mammogram?


Speculated mass + microcalcifications

What other tests to do?


Tissue biopsy (TRU-cut, FNAC) or Excisional)

FNAC done showed C4 lesion what this mean?


• C1 – inadequate sample
• C2 – benign
• C3 – equivocal
• C4 – suspicious
• C5 – malignant

Excision, you are given a pathology report.


What to report/look for?
• Type of cancer
• Number of positive lymph nodes
• Margins status
• HER2 receptors status – poor prognosis
• ER/PR receptor status – good prognosis
• Ki 67 proliferation index – how progressive the cancer is? i.e. the
higher its % the higher the progression of the cancer

What is the most common site of breast cancer?


Most common site is upper outer quadrant as it has more glandular tissue

What’s the most common type of breast cancer?


Invasive duct carcinoma

Anatomic origins of common breast lesions.

18
18 MO’s MRCS B NOTES (Previously Reda’s called Notes) PATHOLOGY
HER2?
Oncogene, biomarker, transmembrane Human Epidermal growth factor Receptor 2 and it is overexpressed in 15% of
breast cancer cases and associated with bad prognosis. They're less likely to be sensitive to hormone therapy
Test: IHC, FISH.

How to test for HER2


Immunohistochemistry (IHC) measures the amount of HER2 protein in the cancer cells.
Fluorescence in situ hybridization (FISH) looks at the number of copies of the HER2 gene in the cancer cells.

Not
important

Identification of HER2-positive breast cancer. HER2 protein overexpression is virtually always caused by amplification of the region of
chromosome 17q that contains the HER2 gene. The increase in HER2 gene copy number is detected by fluorescence in situ
hybridization (FISH) using a HER2-specific probe (red signal), which is typically co-hybridized to tumor cell nuclei with a second probe
specific for the centromeric region of chromosome 17 (green signal), allowing the chromosome 17 copy number to be determined.
Alternatively, HER2 protein overexpression in tumor cells can be detected by immunohistochemical staining with antibodies specific for
HER2.

What is HER positive?


HER2-positive breast cancer is a breast cancer that tests positive for a protein called human epidermal growth factor
receptor 2 (HER2), which promotes the growth of cancer cells.
They're less likely to be sensitive to hormone therapy
HER2-positive breast cancers tend to grow faster and are more likely to spread and come back.
Treatments that specifically target HER2 are very effective. (e.g. Lapatinib, Neratinib, Pertuzumab, Trastuzumab)

Why is it important to know if HER positive or negative?


For treatment choice (see above) and for prognosis

Herceptin (trastuzumab) and how it works at cellular level?


Trastuzumab, (Herceptin) is a monoclonal antibody that interferes with the HER2/neu receptor, which are embedded in
the cell membrane and communicate molecular signals from outside the cell to inside the cell, and turn genes on and off.
The HER (human epidermal growth factor receptor) protein, binds to human EGF and stimulates cell proliferation,
leading to inhibition of MAPK and PI3K-Akt.

Herceptin: (trastuzumab) mode of action?


Causes antibody mediated destruction of cells overproducing HER2, taken 3 times weekly for 12 months.

19
PATHOLOGY MO’s MRCS B NOTES (Previously called Reda’s Notes) 19
What else in management?
• Radiotherapy
• Chemotherapy
• Hormonal therapy:
o Premenopausal: Tamoxifen (20mg /d) for 5 years – blocks estrogen receptor Of androgen
o Postmenopausal: Aromatase inhibitors (anastrozole) to prevent peripheral conversion to estrogen

Who should be involved in care of this patient?


Radiologist, surgeon, oncologist, pathologist.

Patient going for an implant and flap, what single microbiological screening test would you do for this patient?
MRSA screen

What if positive?
Patient is a carrier and will require decolonization – According to trust protocol Please note the
Nose: Mupirocin 2% (Bactroban Nasal®) nasal ointment TDS for 5 days difference in treatment
Skin: Once daily wash with Chlorhexidine 4% (Hibiscrub®) for 5 days between MRSA infection
Hair: Wash with Chlorhexidine 4% (Hibiscrub®) on day 1 and day 5 and MRSA carrier

Now has breast erythema and discharge from nipple, what single microbiological test would you do now?
Cultures and sensitivity

What is the causative organism?


Staph. aureus

How would you treat her?


• Broad spectrum antibiotic (after discussion with microbiologist and according to trust policy) most likely
Flucloxacillin
• Wound care

Patient has redness around nipple, why?


• Eczema
• Paget disease

Pathophysiology of Paget disease?


It is caused by the extension of DCIS (ductal
carcinoma in situ) up the lactiferous ducts and
into the contiguous skin of the nipple,
producing a unilateral crusting exudate over the
nipple and areolar skin.
Unlike Paget disease of the vulva, Paget disease
of the nipple stems from in situ extension of an
underlying carcinoma.

What is the relation between pleomorphic adenoma and breast cancer?


- Mutation of BSG gene that encodes protein which is the native base
for breast cancer and salivary gland expression.

Paget disease of the nipple. Ductal carcinoma in situ arising within the
ductal system of the breast can extend up the lactiferous ducts and into the
skin of the nipple without crossing the basement membrane. The malignant
cells disrupt the normally tight squamous epithelial cell barrier, allowing
extracellular fluid to seep out and form an oozing scaly crust.

20
20 MO’s MRCS B NOTES (Previously Reda’s called Notes) PATHOLOGY
STATION 12 (first stem):
STATION 13 (second stem):
44 year old man seen in clinic with history of mild
asthma and parathyroid surgery, he has had an

What is parathyroid hyperplasia?


MEN 1 Syndrome intermittent abdominal discomfort and pancreatic
disease is being considered
- a non malignant pathology in which there is proliferation of parathyroid cells and enlargement of cells
Define hyperplasia? SCENARIO:
Increase in the number of cells in tissue or organ in response to a stimulus Man with parathyroidectomy,
pancreatic mass
How many parathyroid glands are mostly affected?
The 4 glands Define the two top reasons for parathyroid hyperplasia?
- Renal failure
- MEN syndrome
Microscopic picture of hyperplasia?
• Microscopically, the most common pattern seen is that of chief cell hyperplasia, which may involve the glands in
a diffuse or multinodular pattern.
• Less commonly, the constituent cells contain abundant water-clear cells (“water-clear cell hyperplasia”).
• In many instances there are islands of oxyphils, and poorly developed, delicate fibrous strands may envelop the
nodules.

The patient developed stupor, confusion and hypoglycemia (1.2 mmol/l), why do you think that happened?
Insulinoma

What cell derived from?


β-cell of islets of Langerhans’

What other causes of unresponsive hypoglycemia do you know?


• Abnormal insulin sensitivity,
Q: Apart from insulinoma which produces insulin, What other pathological process can create similar hypoglycemia?
• Diffuse liver disease, - Insulin like growth factors which produced from non-pancreatic tumors.
• Inherited glycogenoses,
• Ectopic production of insulin by certain retroperitoneal fibromas and fibrosarcomas

Clinical picture of Insulinoma?


• Confusion
• Stupor
• Loss of consciousness (blood glucose 2.5 mmol/L or less)
These episodes are precipitated by fasting or exercise
Promptly relieved by feeding or parenteral administration of glucose.

Biochemical diagnosis: Q: How the diagnosis of insulinoma is confirmed biochemically?


High circulating levels of insulin (>10 μU/mL) - By finding high detectable insulin levels in blood stream at the same time hypoglycemia is occurring.
- High insulin-to-glucose ratio
• High insulin-to-glucose ratio - High level of C-peptide

• High level of C-peptide
What do you suspect as another pathology in this patient?
Imaging for diagnosis?
Pituitary adenoma (MEN 1) -US
. -CT
What are the 3 gene mutations in Insulinoma? -MRI
1- MEN1, which causes familial MEN syndrome, type 1, also is mutated in a number of sporadic neuroendocrine
tumors
2- Loss-of-function mutations in tumor suppressor genes such PTEN and TSC2 (which results in activation of the
oncogenic mammalian TOR (mTOR) signaling pathway
3- Inactivating mutations in two genes, alpha-thalassemia/mental retardation syndrome, X-linked (ATRX) and
death-domain associated protein (DAXX), which have multiple cellular functions, including telomere
maintenance.

What is two hit hypothesis?


Like all genes, tumor suppressor genes may undergo a variety of mutations; however, most loss-of-function mutations
that occur in tumor suppressor genes are recessive in nature. Thus, in order for a particular cell to become cancerous,
both of the cell's tumor suppressor genes must be mutated. This idea is known as the "two hit" hypothesis
Q: Define MEN?
What is telomere? - Group of related conditions inherited as autosomal dominant characterized by hyperplasia or neoplasia of the endocrine organs.
A telomere is a region of repetitive nucleotide sequences at each end of a chromosome, which protects the end of the
chromosome from deterioration or from fusion with neighboring chromosomes
Q: What are the genes responsible for cancer in general?
- Tumor suppressor genes
What is apoptosis? - Proto-oncogenes
Programmed cell death - micro-RNA genes
- Mutatory genes: like BRCA1 & 2

22
22 MO’s MRCS B NOTES (Previously Reda’s called Notes) PATHOLOGY
See Gangrene / Mesothelioma station

STATION 6:

23
PATHOLOGY MO’s MRCS B NOTES (Previously called Reda’s Notes) 23
MEN 2 Syndrome

Single best test to diagnose? SCENARIO:


FNAC Female with thyroid nodule,
elevated calcitonin levels
Pathology report: FNAC showed malignant cell features, amyloid deposits,
immunohistochemistry stains positive with calcitonin, stains negative for thyroxine

What is the type of cancer?


Medullary thyroid cancer

Why?
• Amyloid deposits
• Calcitonin positive on IHC

Cell source?
Parafollicular C cells
What is the best way to know the origin of the cells? IHC
What is IHC in simple words?
It is a method of localizing specific antigens in tissues or cells based on antigen antibody recognition

How does it work?


The antibodies are usually linked to an enzyme or a fluorescent dye. When the antibodies bind to the antigen in
the tissue sample, the enzyme or dye is activated, and the antigen can then be seen under a microscope.

What is the type of Ag- Ab reaction in IHC?


Complement fixation

Another pathology report showing size, number of positive lymph nodes (2-6)

Do TNM (In the exam will give you size and regional LN and mets or not and then ask to do TNM)

Tumor
Tx Primary cannot be assessed
T0 No evidence of primary
T1 Limited to thyroid, 1 cm or less
T2 Limited to thyroid > 1 cm but < 4 cm
T3 Limited to thyroid > 4 cm
T4 Extending beyond capsule, any size
Notes
Nx Cannot be assessed
N0 No regional lymph node metastases
N1 Regional node metastases
M status
Mx Cannot be assessed
M0 No metastases
M1 Metastases present

Stage Under 45 years Over 45 years


I Any T, any N, M0 T1, N0, M0
II Any T, any N, M1 T2, N0, M0 or T3, N0, M0
II T4, N0, M0 or any T, N1, M0
IV Any T, any N, M1

If the patient developed hypertension, what do you think she might have?
Pheochromocytoma Q: Other adrenal gland swelling?
- infection: abscess
- trauma: hematoma
- neuroblastoma
- lymphangioma
24
24 MO’s MRCS B NOTES (Previously Reda’s called Notes) PATHOLOGY
How can you diagnose pheochromocytoma?
Plasma:
• Free metanephrine
• Catecholamines
• Catecholamines stimulation test
Urine:
• Fractionated metanephrine
• Total metanephrine
• Catecholamines
• VMA (Veneline Mandelic Acid)
Imaging:
• Ultrasound
• CT
• MRI
• MIBG scintigraphy
• PET/CT

If this condition was familial, what other condition you suspect?


Parathyroid hyperplasia

Types of MEN syndromes? Oncogene mutation? Mode of inheritance?


The table below summarizes the three main types of MEN:
MEN type I MEN type IIa MEN type IIb
Mnemonic 'three P's': • Hyperparathyroidism (usually Same as MEN IIa with addition of:
• Parathyroid (95%): Parathyroid adenoma hyperplasia) • Marfanoid body habitus
• Pituitary (70%): Prolactinoma/ACTH/Growth • Pheochromocytoma • Mucosal neuromas
Hormone secreting adenoma • Medullary thyroid cancer
• Pancreas (50%): Islet cell tumors/Zollinger
Ellison syndrome
also: Adrenal (adenoma) and thyroid
(adenoma)

Most common presentation = hypercalcemia


MENIN gene (chromosome 11) RET oncogene (chromosome 10) RET oncogene (chromosome 10)
Autosomal Dominant Autosomal Dominant Autosomal Dominant

Treatment of medullary thyroid cancer? Q: Types of neck dissection?


Total thyroidectomy with block neck dissection - Radical
After doing full examination and investigation and staging - Modified radical (spare IJV, SCM, & CN11)
I will discuss with a MDT to go for Total thyroidectomy with
block neck dissection if +ve nodes. - Selective

25
PATHOLOGY MO’s MRCS B NOTES (Previously called Reda’s Notes) 25
STATION 8:
Parotid Tumors

SCENARIO – 1:
65-year-old man with a 2-month history of unilateral 4 cm parotid swelling without facial nerve problem

See Parotid Gland, Head & Neck Anatomy


See Head & Neck Surgery file, Part A Notes
What is the most common parotid benign swelling?
Pleomorphic adenoma

What is the meaning of pleomorphic?


Remarkable histologic diversity

Describe pleomorphic adenoma appearance? Variety of histological appearance reflecting stromal and epithelial and myoepithelial elemnts.
Benign tumors that consist of a mixture of ductal (epithelial) and myoepithelial cells, and therefore they show both
epithelial and mesenchymal differentiation

What are the causes of bilateral swelling?


Local Systemic Drugs
• Mumps • Sarcoidosis • High oestrogen OCP
• Parotitis • TB • Thiouracil
• Sialectasis • Alcoholism/liver cirrhosis • Isoprenaline
• Sjogren’s • Cushing’s
• Neoplasia • Bulimia Nervosa

What are the causes of unilateral parotid swelling?


• Duct obstruction – salivary calculus, external ductal compression
• Neoplasia – benign or malignant
• Infective mumps (although bilateral swelling is more common), parotitis

Types of parotid neoplasm?


Benign (85%) Malignant (15%)
Pleomorphic adenoma (mixed) (50%) Mucoepidermoid carcinoma (15%)
Warthin tumor (5% - 10%) Adenocarcinoma (NOS) (10%)
Oncocytoma (1%) Acinic cell carcinoma (5%)
Other adenomas (5% - 10%) Adenoid cystic carcinoma (5%)
Basal cell adenoma Malignant mixed tumor (3%-5%)
Canalicular adenoma Squamous cell carcinoma (1%) Pleomorphic adenoma.Low-power view
showing a welldemarcated tumor with
Ductal papillomas Other carcinomas (2%)
adjacent, deeply staining, normal salivary
(With 4 marks) gland parenchyma.
What are the clinical signs of malignant parotid tumour?
• Facial nerve affection
• Rapid increase in size
• Fixity to underlying tissue
• Invasion of the skin
• Skin ulcer
• Presence of associated nodes

What are the characteristics of malignant neoplasms?


• More rapid increase in size
• Less differentiation (or lack of differentiation, called anaplasia)
• Tendency to invade surrounding tissues (not respect tissue boundaries)
• Ability to metastasize to distant tissues
Pleomorphic adenoma. High-power view
Q; Can pleomorphic adenoma turn to malignant tumor? showing epithelial cells as well as
Yes, in 5% of people in process called carcinoma Ex-polymotphic adenoma myoepithelial cells within chondroid
Due to accumulation of mutation. matrix material.

If you consider the likely cytology feature of polymorphic adenoma, what will raise concern for malignancy from the tumor? (3 marks)
- Atypical polymorphic cell forms
- Increased atypical mitosis
- Necrosis or hemorrhage
26
26 MO’s MRCS B NOTES (Previously Reda’s called Notes) PATHOLOGY
Features of malignant cells? Cytologic features of malignant neoplasms
• Invasion: Malignant cells do not respect tissue boundaries, and • Increased nuclear size (with increased
can be seen infiltrating or invading into surrounding structures nuclear/cytoplasmic ratio--N/C ratio).
• Increased mitotic rate: Malignant cells will often have increased • Variation in nuclear or cell size
numbers of mitoses. (pleomorphism).
• Differentiation and anaplasia: Normal cells are usually • Lack of differentiation (anaplasia).
structured in a particular way that corresponds with their • loss of normal tissue architecture (loss
function. This is known as differentiation. Malignant cells may of polarity)
become less differentiated as part of their path to malignancy. • Increased nuclear DNA content with
This is known as anaplasia. Well differentiated malignant cells subsequent dark staining on H and E
show features similar to the parent tissue. For example, well slides (hyperchromatism).
differentiated adenocarcinoma cells will tend to form gland-like
• Prominent nucleoli or irregular
structures; well differentiated squamous cell carcinomas may
chomatin distribution within nuclei.
show intercellular bridging or keratin formation. Poorly
• Increased mitotic rate (especially
differentiated cells have lost most of their resemblance to the
irregular or bizarre mitoses).
parent tissue, which may be difficult to identify without special
• Giant cells some malignant cells may
staining techniques. Anaplastic cells have no resemblance to
coalese into so-called giant cell
their parent tissue, and usually indicate a very aggressive
• Ischemic necrosis (from tumor cells
malignancy.
outgrowing their blood supply)
Anaplastic Features
o Loss of normal tissue architecture: Normal cells are
usually arranged in an orderly fashion. Epithelial cells often have polarity, with their nuclei at a
specific location. Malignant cells lose this architecture and are arranged haphazardly
o Pleomorphism: Malignant cells may show a range of shapes and sizes, in contrast to regularly sized
normal cells. The nuclei of malignant cells are often very large (often larger than the entirety of a
normal cell) and may contain prominent nucleoli.
o Hyperchromatic nuclei: The nuclei of malignant cells typically stain a much darker colour than their
normal counterparts.
o High nuclear-cytoplasmic ratio: The nuclei of malignant cells often take up a large part of the cell
compared with normal cell nuclei
o Giant cells: Some malignant cells may coalesce into so-called giant cells, which might contain the
genetic material of several smaller cells.

Anaplasia?
Lack of differentiation (loss of similarity to the mother cell indicate very aggressive tumor)
What type of pathological invx at the time of first clinic appointment must be done?
Single best test to differentiate between benign and malignant cells?
FNAC

Difference between cytology and histology?


Cytology is the study of cellular structure and function
Histology is the study of tissue under the microscope

How to differentiate between carcinoma and lymphoma?


IHC - See Breast Cancer station and MEN2 syndrome station

How to rule out malignancy intraoperative?


Frozen section – See Peptic Ulcer Disease + Hyperparathyroidism station

FNAC findings
• If you find Langerhans giant cell + lymphocytes + necrotic material → granuloma (TB)
• If you find Brown pigmented cell + epithelioid cells → malignant melanoma Malignant epitheliod cells with bizrre neuclei and brown pigment
in cytoplasm.
• If you find Reed Sternberg cell + lymphoid cell + blast cell → lymphoma(Polymorphic lymphoid cells with mitosis and blast elements)

What is the role of B-catenin in pleomorphic adenoma?


B-catenin play an important role in WNT pathway and the complex formed by this molecule play critical role in maintaining normal growth of cells,
any alteration of this components will lead to loss of cell to cell adhesion and contribute to carcinogenisis.

27
PATHOLOGY MO’s MRCS B NOTES (Previously called Reda’s Notes) 27
FNAC done and you have needle stick injury, what you will do?
• The wound should be allowed to bleed under running water and wash with soap
• Assess incident risk
• Assess source patient
o Take history if he has a possibility of any blood borne infection, take a blood sample from the patient if
high risk after consent.
• I would file an incident report and speak to occupational health for advice.
• Documentation
Source: https://www.bmj.com/content/351/bmj.h3733

28
28 MO’s MRCS B NOTES (Previously Reda’s called Notes) PATHOLOGY
What are the postoperative complications of Sweat glands
parotidectomy? (Damage of 3 nerves + fistula)
• 7th CN palsy
• Frey’s Syndrome
Sympathetic
• Salivary fistula
• Greater auricular nerve damage -numbness to Sensory
earlobe Parasympathetic

What is Frey’s Syndrome?


Auriculotemporal syndrome: post-operative A Parotid
phenomena following salivary gland surgery resulting in
gustatory sweating and facial flushing due to Sweat glands
reinnervation of postganglionic fibers to sweat gland and
cutaneous blood vessels Mixed
Sympathetic
After 10 years from resection patient develop neck
swelling, cause?
Parasympathetic Parotid
Possible recurrence with metastasis Mixed bed
- Lymphoma
- carotid body tumor B
What is the definition of a high sensitivity test?
Frey's syndrome. A, Normal anatomy. B, Mixing of fibers dt surgery.
Sensitivity is the ability of a test to correctly classify an
truly
individual as ′diseased′ (true positive rate)
High sensitivity = low number of false negatives results when applied to test population.

What is the definition of a high specificity test?


Specificity is the ability of a test to correctly classify an individual as disease-free (true negative rate).
High specificity = low number of false positives results when applied to test population.

Screening Test Statistics


Disease present Disease absent
Test positive TP FP
Test negative FN TN
TP = true positive; FP = false positive; TN = true negative; FN = false negative

Sensitivity TP / (TP + FN ) Proportion of patients with the condition who have a


positive test result
Specificity TN / (TN + FP) Proportion of patients without the condition who have a
negative test result
Positive predictive value TP / (TP + FP) The chance that the patient has the condition if the
diagnostic test is positive
Negative predictive value TN / (TN + FN) The chance that the patient does not have the condition
if the diagnostic test is negative
Likelihood ratio for a sensitivity / (1 - specificity) How much the odds of the disease increase when a test
positive test result is positive
Likelihood ratio for a (1 - sensitivity) / specificity How much the odds of the disease decrease when a test
negative test result is negative
Positive and negative predictive values are prevalence dependent. Likelihood ratios are not prevalence dependent

29
PATHOLOGY MO’s MRCS B NOTES (Previously called Reda’s Notes) 29
STATION 1:
SCENARIO – 2:
55 years old female with Mastectomy done 5 years back and parotidectomy for pleomorphic adenoma 10 years back.+ LN enlargement
Now
Why presented
this by neck swelling (supra-clavicular LN swelling)
pt has lymphadenopathy?
- Due to metastasis
What is the relation between breast cancer and pleomorphic adenoma?
Genetic (BRCA1 and BRCA2)
The is a genetic relationship between them in their development
expressing BSG gene
Investigation?
FNAC or Tru-Cut® biopsy

What is the advantage of FNAC over Tru-Cut biopsy?


FNAC Tru-Cut
• Cost effective • Avoid unnecessary excisional biopsy
Advantage

• Simple • More reliable


• Quick result • Grading and typing of factor
• Less invasive • Relatively easy
• False -ve • More invasive than FNAC
Disadvantage

• Diagnosis is based on purely cytological evaluation • Expensive


• Insufficient sample • Scar
• Inability to differentiate invasive from in situ
carcinoma

Fine needle aspiration cytology


(FNAC) is an operator dependent
procedure that may or may not be
image guided and essentially
involves passing a needle through
a lesion whilst suction is applied to
a syringe. The material thus
obtained is expressed onto a slide
and sent for cytological
assessment.

Tissue samples may also be


obtained by both core and tru cut
biopsy. A core biopsy is obtained
by use of a spring-loaded gun with
a needle passing quickly through
the lesion of interest. A tru cut
biopsy achieves the same objective
but the needle moved by hand.
FNAC Needle biopsy

What test to be done in FNAC?


IHC, receptor status - See Breast Cancer station and MEN2 syndrome station

What is the meaning of pleomorphic?


Describe pleomorphic adenoma appearance? Variety of histological appearance reflecting stromal and epithelial and myoepithelial elemnts.
After 10 years from resection patient develop neck swelling what this indicate?
- Possible recurrence with metastasis, lymphoma, Carotid body artery
What other things do you consider? What is DD?
• Lymphoma
• Carotid body tumor
• Carcinoma

If the metastasis from the erodes carotid artery it doesn’t stop bleeding, why?
Because of high blood flow and arterial wall may be necrotic (carotid blowout syndrome), and artery lose its ability to
undergo vasoconstriction in response ti injury

30
30 MO’s MRCS B NOTES (Previously Reda’s called Notes) PATHOLOGY
Define metastasis:
Survival and growth of cells at a site distant from their primary
origin

List 5 pathological steps by which carcinoma spreads?


• Direct or continuous extension
• Penetration into lymphatic, blood vessels or body
cavities
• Transport of the tumour cells into the circulation
• Arrest in the capillary beds of secondary sites
• Growth of the disseminated tumour cells in
secondary sites

The metastatic cascade. Sequential steps involved in the


hematogenous spread of a tumor.

Main routs of spread of malignant tumor?


• Lymphatic (permeation and lymph emboli)
• Hematogenous
• Trans-celomic (along body cavity to peritoneal
surface)

How or what is the mechanism of lymphatic spread?


• As the cancer cells and the tumor -associated
macrophages secrets growth factors such as VEGF-C
and VEGV-D to induce lymph-angiogenesis in primary
Invasion of the ECM initiates the metastatic cascade and is an
active process that can be resolved into several steps tumor and in draining sentinel LN, thereby promoting
• “Loosening up” of tumor cell–tumor cell interactions LN metastasis
• Degradation of ECM • Also, primary tumors release immunomodulatory
• Attachment to novel ECM components molecules like exosomes which lead to
• Migration and invasion of tumor cells immunosuppression

31
PATHOLOGY MO’s MRCS B NOTES (Previously called Reda’s Notes) 31
STATION 2:
Nasopharyngeal Carcinoma

Define carcinoma?
Carcinoma is a type of cancer that develops from epithelial cells.

What are the differences between Benign and malignant cells?


Benign and malignant tumors can be distinguished from one another based on the degree of differentiation, rate of
growth, local invasiveness, and distant spread.
• Benign tumors resemble the tissue of origin and are well differentiated; malignant tumors are poorly or
completely undifferentiated (anaplastic).
• Benign tumors are slow-growing, whereas malignant tumors generally grow faster.
• Benign tumors are well circumscribed and have a capsule; malignant tumors are poorly circumscribed and
invade the surrounding normal tissues.
• Benign tumors remain localized to the site of origin, whereas malignant tumors are locally invasive and
metastasize to distant sites.

Q: What is the mechanism of radiotherapy?


Ionizing radiation works by damaging the DNA of
cancerous (base damage, single and double-strand
breaks, and crosslinks between DNA and protein)
tissue leading to cellular death.

What is the SI units of radiotherapy? (Asked about dose only)


• Coulomb/kg: Exposure
• Gray (Gy): Dose Exposure (Absorbed dose)
• Sievert (Sv): Dose equivalent
• Becquerel: Activity

Risk factors for nasopharyngeal carcinoma? 2.


• Gender: twice as often in males as it is in 1.
5.
females. 3.
• Ethnicity and race: most common in 4.

southern China (including Hong Kong),


Singapore, Vietnam, Malaysia, and the
Philippines. It is also fairly common in
Northwest Canada and Greenland.
• Diet: consumption of salted fish
containing carcinogenic volatile
nitrosamines
• Infection with the Epstein-Barr virus Effects of ionizing radiation on DNA and its consequences. The effects on DNA
• Genetic factors can be direct, or most importantly, indirect, through free radical formation.
• Family history
• Tobacco and alcohol use Gray
It is defined as the absorption of one joule of
Patient had an oral lesion, swab showing hyphae? Fungal infection radiation energy per kilogram of matter
Mainly Candida
Sievert
The sievert takes into account the relative
Q: What are the risk factors for that patient for oral candidiasis?
biological effectiveness (RBE) of ionizing
• As a complication of radiotherapy or chemotherapy radiation, since each form of such radiation—
• Being a diabetic e.g., X-rays, gamma rays, neutrons—has a
slightly different effect on living tissue.
Q: What are the routes of spread of nasopharyngeal carcinoma? Accordingly, one sievert is generally defined as
the amount of radiation roughly equivalent in
• Lymphatic spread
Deferential diagnosis? biological effectiveness to one gray
• Local invasion - Adenoid hypertrophy
- infection: abscess Becquerel
- Hematoma
What are other non-epithelial tumor can be here? One becquerel is defined as the activity of a
#Tumors:
- Lymphoma quantity of radioactive material in which one
- Angiofibroma
- Rhabdomyosarcoma nucleus decays per second. The becquerel is
- lymphoma
- Angiofibroma
- Rhabdomyosarcoma therefore equivalent to an inverse second, s−1.

32
32 MO’s MRCS B NOTES (Previously Reda’s called Notes) PATHOLOGY
Where to spread locally?

1-

2-

6-
5-
7-

8-

9-

3-

4-

What are the common lymph node tumors?


• Lymphoma (Hodgkins and Non-Hodgkins)
• Leukemia
• Metastatic

How will you investigate for metastasis in lymph nodes?


• Tissue biopsy: FNAC, Excisional biopsy, sentinel biopsy for histopathological assessment with or without
immunohistochemistry
• Imaging modalities: ranging from ultrasound scan, CT scan, MRI and PET-CT

33
PATHOLOGY MO’s MRCS B NOTES (Previously called Reda’s Notes) 33
STATION 3:
Carcinoid Tumor

SCENARIO:
Middle aged man with RIF pain, surgery was done, revealed dilated appendix (looks like -mass), histopathology
revealed appendicular abscess plus 6mm appendicular mass involving the mucosa and the muscularis layers

Define abscess?
Abscess is focal collection of pus that may be caused by seeding of pyogenic organisms into a tissue or by secondary
infections of necrotic foci.

Structure of the abscess?


Abscess typically has a central, largely necrotic region rimmed by a layer of preserved neutrophils, with a surrounding
zone of dilated vessels and fibroblast proliferation indicative of attempted repair

How neutrophils migrate to the site of inflammation?


See Diverticulitis + Endometriosis station
The neutrophils first roll, then become activated and adhere to endothelium, then transmigrate across the endothelium,
pierce the basement membrane, and migrate toward chemoattractants emanating from the source of injury.

Blood tests to identify the inflammation? How to know the causative organism clinically?
• ESR, CRP From color of pus
• White blood cells count - yellow most likely s. aurias
- green most likely pseudomonas
- silver granules most likely actinomycetes
What is carcinoid tumor?
Slow-growing type of neuroendocrine tumor originating in the cells of the neuroendocrine system.

Where it is most commonly found?


Small intestine, appendix (terminal third) and they can also be found in the rectum, stomach and lung

Cells arising from?


Enterochromaffin (EC) cells (also known as Kulchitsky cells) in the crypts of Lieberkuhn

What does it release?


Notably: serotonin (5-HT), bradykinins, prostaglandins, tachykinins, substance P and histamine

Clinical presentation?
• Periodic abdominal pain
• Manifestations of carcinoid syndrome:
o Cutaneous flushing
o Diarrhea and malabsorption
o Cardiac manifestations: Valvular heart lesions, fibrosis of the endocardium
o Wheezing or asthma like syndrome: Due to bronchial constriction

How does it spread?


Cells produce a significant amount of beta-catenin, which enables the tumor cell adhesion, thus promoting metastasis.

Most common metastatic site?


The liver

Why a person with heavy metastasis in liver has symptoms than with primary tumor?
That's because most of the blood circulation from the gastrointestinal tract must pass through the liver before it reaches
the rest of the body. The liver has strong enzymes that break down and neutralize most of the excess serotonin and
other substances produced by the carcinoid tumors, preventing them from reaching tissues where they can cause
symptoms. When carcinoid tumors metastasize to the liver, the substances they overproduce can more easily reach the
bloodstream, and reach tissues where they can cause symptoms.

Diagnosis of carcinoid syndrome?


• Chromogranin A (CgA) Testing in the blood (protein secreted from carcinoid tumor cells)
• 5-Hydroxyindoleacetic Acid (5-HIAA) Testing in 24 h urine (byproduct of serotonin)
• Pathological diagnosis: immunohistochemistry stains positive for chromogranin B
This page intentionally left blank

34
34 MO’s MRCS B NOTES (Previously Reda’s called Notes) PATHOLOGY
STATION 9:
Inflammatory Bowel Disease (IBD)

What is ulcerative colitis? SCENARIO: A Lady known to have


Is chronic inflammatory disease involving all or part of the colon ulcerative colitis and on surveillance
characterized by inflammatory process confined to mucosa and nearly colonoscopy is found to have a lesion
always involves the rectum. less than 1cm in sigmoid colon.
Pathogenesis?
Almost Idiopathic Maybe immunological, environmental or genetic

Colonoscopy done with biopsy showing tubular dysplasia in one part, adenocarcinoma in other part showing a picture of a
tumor eroding through the muscularis layer + 1/4 positive node

What will you offer this lady? Total colectomy Proctocollectomy with end ileostomy
Why? The whole colon is susceptible

If there is liver Mets, how will this affect TNM staging?


M1
Backwash ileitis? Involvement of terminal ileum due to longstanding UC
with no skip lesions
Microscopic features of Crohn’s?
See table below

Microscopic features of UC?


See table below

UC Complications?
• Intestinal obstruction Clinical features of UC?
• Fistula formation - Abdominal pain
- diarrhea
• Abscess - bleeding per rectum
• Toxic megacolon - extra intestinal manifestation
• Malabsorption
• Malignancy
• Gall stones (due to inhibition of enterohepatic circulation so bile salts will not be absorbed leading to increased
amount of cholesterol)
Asked 2 skin and 2 eye
Extraintestinal manifestation of
Why is the patient having diarrhea? manifestation inflammatory bowel disease: A PIE SAC
• Malabsorption • Aphthous ulcers
• Infection • Pyoderma gangrenosum
• Increased motility • Iritis, Conjunctivitis, episcleritis, anterior uveitis
• Erythema nodosum
Why you need endoscopic surveillance? • Sclerosing cholangitis
For risk of colon cancer • Arthritis
• Clubbing
Renal stone formation in Crohn’s?
Increased intestinal fat (due to malabsorption) → binds to calcium → leaving oxalates (hyperoxaluria)

Had resection of terminal ileum but diarrhea continued, possible cause?


Relapse of Crohn’s or malabsorption Short bowl syndrome
Management of Crohn’s (Mainly medical)
Stoma → ischemia, what to do? Medical Second:
Inform consultant, patient relatives, consider refashioning • Advice from gastroenterology
• Steroid, antibiotics, 5-aminosalicylic
Type of vitamin deficiency? acid, immunomodulators
A, D, E, K deficiency and B12 Conservative First life style modification
• Dietary control (low residue diet)
Other investigations? Surgery for (Limited resection)
• Stool analysis (For Calprotectin & fecal ocult blood) • Refractory disease
• Barium follow through X-ray, CT • Intestinal obstruction
• Prothrombin concentration: to detect vit. K def. • Toxic megacolon
• Calcium oxalate levels • Abscess, fistula, perforation,
• Full blood count: macrocytic anemia hemorrhage, cancer
- Colonoscopy but not in acute phase

35
PATHOLOGY MO’s MRCS B NOTES (Previously called Reda’s Notes) 35
Crohn's disease Ulcerative colitis
Common Site Terminal ileum Rectum
Distribution Mouth to anus Rectum and colon (back-wash ileitis)
Macroscopic • Cobblestone appearance • Strictures • Pseudopolyps
changes and • Aphthoid ulceration • Extensive ulceration (Superficial)
gross features • Linear fissures • Contact bleeding
• Thickened bowel wall
• Creeping fat (Not in colonoscopy)
Depth of disease Transmural inflammation Superficial inflammation
Dist. pattern Patchy, skip lesions Continuous
Histological Granulomas (non caseating epithelioid cell aggregates Crypt abscesses, Inflammatory cells in
and with Langhans' giant cells) the lamina propria
microscopic
features Active colitis Active colitis
• Cryptitis • Cryptitis
• Crypt abscess formation • Crypt abscess formation
• Mucosal ulceration • Mucosal ulceration
• Inflammatory pseudo polyps • Inflammatory pseudo polyps
• Inflammation involves all wall layers
(Transmural)
Chronic colitis Chronic colitis
• Increased lymphoplasmacytic lamina propria • Increased lymphoplasmacytic lamina
infiltrate propria infiltrate
• Paneth cell metaplasia – left colon • Paneth cell metaplasia in left colon
• Pseudopyloric gland and gastric surface • Architectural distortion
(foveolar) metaplasia o Branched, dilated irregular
• Paneth cell hyperplasia – small bowel and crypts
right colon o Crypts don’t reach the
muscularis mucosa

Extraintestinal Uncommon More common


Manifestations
Cancer Risk Slightly 1-3% 5-25%

36
36 MO’s MRCS B NOTES (Previously Reda’s called Notes) PATHOLOGY
Describe adenoma carcinoma sequence?
Stepwise accumulation of mutations of oncogenes and tumor suppressor genes:
1- Loss of APC (tumor suppressor gene) → hyperplasia
2- K-RAS (proto-oncogene) mutation → dysplasia
3- Loss of p 53 (tumor suppressor gene) → adenocarcinoma

Molecular model for the evolution of colorectal cancers through the adenoma-carcinoma sequence. Although APC mutation is an early
event and loss of TP53 occurs late in the process of tumorigenesis, the timing for the other changes may be variable. Note also that
individual tumors may not have all of the changes listed. Top right, cells that gain oncogene signaling without loss of TP53 eventually
enter oncogene-induced senescence. LOH, loss-of-heterozygosity.

How proto-oncogenes and tumor suppressor genes


act?
Genes that promote autonomous cell growth in
cancer cells are called oncogenes, and their
unmutated cellular counter-parts are called proto-
oncogenes. Oncogenes are created by mutations in
proto-oncogenes and encode proteins called
oncoproteins that have the ability to promote cell
growth in the absence of normal growth-promoting
signals. Oncoproteins resemble the normal products
of proto-oncogenes but bear mutations that are
often inactivate internal regulatory elements;
consequently, their activity in cells does not depend
on external signals. Cells expressing oncoproteins
are thus freed from the normal checkpoints and
controls that limit growth, and as a result proliferate
excessively
• Proto-oncogenes: normal cellular genes
whose products promote cell proliferation
• Oncogenes: mutated or overexpressed
versions of proto-oncogenes that function
autonomously, having lost dependence on
normal growth promoting signals
• Tumor suppressor genes (p53, APC):
normal genes whose absence can lead to
development of cancer, they act as
gatekeepers: inhibit proliferation or
promote the death of cells with damaged
DNA

Growth factor signaling pathways in cancer. Growth factor receptors,


RAS, PI3K, MYC, and D cyclins are oncoproteins that are activated by
mutations in various cancers. GAPs apply brakes to RAS activation,
and PTEN serves the same function for PI3K.

37
PATHOLOGY MO’s MRCS B NOTES (Previously called Reda’s Notes) 37
Function of KRAS? ‫بجيب بروتني شغلتو يوصل رسايل من برا الخلية لي النواة فيها تعليمات بتقول لي الخلية اتقسمي وال اتميزي‬
The KRAS gene provides instructions for making a protein called K-Ras that is part of a signaling pathway known as the
RAS/MAPK pathway. The protein relays signal from outside the cell to the cell's nucleus. These signals instruct the cell to
grow and divide (proliferate) or to mature and take on specialized functions (differentiate). The K-Ras protein is a
GTPase, which means it converts a molecule called GTP into another molecule called GDP. In this way the K-Ras protein
acts like a switch that is turned on and off by the GTP and GDP molecules. To transmit signals, it must be turned on by
attaching (binding) to a molecule of GTP. The K-Ras protein is turned off (inactivated) when it converts the GTP to GDP.
When the protein is bound to GDP, it does not relay signals to the cell's nucleus.

Not important Intrinsic and Extrinsic Pathways of apoptosis


Intrinsic and extrinsic pathways of apoptosis
and mechanisms used by tumor cells to evade
cell death. (1) Loss of p53, leading to reduced
function of pro-apoptotic factors such as BAX.
(2) Reduced egress of cytochrome c from
mitochondria as a result of upregulation of
anti-apoptotic factors such as BCL2, BCL-XL,
and MCL-1. (3) Loss of apoptotic peptidase
activating factor 1 (APAF1). (4) Upregulation
of inhibitors of apoptosis (IAP). (5) Reduced
CD95 level. (6) Inactivation of death-induced
signaling complex. FADD, Fas-associated via
death domain.

38
38 MO’s MRCS B NOTES (Previously Reda’s called Notes) PATHOLOGY
Function of P53?
DNA damage and other stress signals may trigger the increase of p53 proteins, which have three major functions: growth
(Arrest arrest, DNA repair and apoptosis (cell death). The growth arrest stops the progression of cell cycle, preventing replication
at G1) of damaged DNA. During the growth arrest, p53 may activate the transcription of proteins involved in DNA repair.
Apoptosis is the "last resort" to avoid proliferation of cells containing abnormal DNA.

The role of p53 in maintaining the integrity of the genome. Activation of normal p53 by DNA-damaging agents or by hypoxia leads to
cell cycle arrest in G1 and induction of DNA repair by transcriptional upregulation of the cyclin-dependent kinase inhibitor CDKN1A
(encoding the cyclin-dependent kinase inhibitor p21) and the GADD45 genes. Successful repair of DNA allows cells to proceed with the
cell cycle; if DNA repair fails, p53 triggers either apoptosis or senescence. In cells with loss or mutations of the p53 gene, DNA damage
does not induce cell cycle arrest or DNA repair, and genetically damaged cells proliferate, giving rise eventually to malignant
neoplasms.

39
PATHOLOGY MO’s MRCS B NOTES (Previously called Reda’s Notes) 39
Function of APC? β-catenin ‫ و بمشو بكسرو جسر‬comlex ‫ النو بتلم مع ناس بعمل ليهو‬WNT ‫بقفل شارع‬
Encodes a factor that negatively regulates the WNT pathway in colonic epithelium by promoting the formation of a
complex that degrades β-catenin

The role of APC in regulating the stability and function of β-catenin. APC and β-catenin are components of the WNT signaling
pathway. A, In resting colonic epithelial cells (not exposed to WNT), β-catenin forms a macromolecular complex containing the APC
protein. This complex leads to the destruction of β-catenin, and intracellular levels of β-catenin are low. B, When normal colonic
epithelial cells are stimulated by WNT molecules, the destruction complex is deactivated, β-catenin degradation does not occur, and
cytoplasmic levels increase. β-catenin translocates to the nucleus, where it binds to TCF, a transcription factor that activates genes
involved in cell cycle progression. C, When APC is mutated or absent, as frequently occurs in colonic polyps and cancers, the
destruction of β-catenin cannot occur. β-catenin translocates to the nucleus and coactivates genes that promote entry into the cell
cycle, and cells behave as if they are under constant stimulation by the WNT pathway.

Q:
It is a proinflammatory cytokine
secreted from macrophages, involved in
systemic inflammation and in making up
the acute phase reaction.
W
h
a
t
i
s
T
N
F
Q: Role of TNF in IBD? ‫وسع وسع كدا يا ابيثيليم‬
TNF and other immune mediated signals
direct epithelia to increase tight junctions
permeability which increase the flux of
luminal bacterial components which
activates innate and adaptive immune
responses

Q: What drugs antagonize TNF?


• Infliximab
• Adalimumab ‫عدالي‬
• Certolizumab ‫سيرتولي‬

Q: Mechanism of action of these drugs?


Monoclonal IgG1 antibody to TNF- α

Why are they used in treatment of UC?


They are called (biologics), used mainly in
steroid refractory UC cases and for
treatment of extra intestinal
manifestations A model of pathogenesis of inflammatory bowel disease (IBD). Aspects of both
They block the inflammatory process Crohn disease and ulcerative colitis are shown.
responsible for the condition.

40
40 MO’s MRCS B NOTES (Previously Reda’s called Notes) PATHOLOGY
STATION 3:
Familial Adenomatous Polyposis (FAP)

SCENARIO: A young lady with endometriosis concerned that her father died of a cancer at an early stage. She had a
colonoscopy just now

Identify a picture of FAP?


Multiple polyps that varies in size and ulceration.

Definition of FAP?
Autosomal dominant condition characterized by loss
of APC tumor suppressor gene on the long arm of
chromosome 5 leading to development of hundreds
of tubular adenomas with 100% risk of cancer by the
age of 40

Function of APC tumor suppressor gene?


Encodes a factor that negatively regulates the WNT
pathway in colonic epithelium by promoting the
formation of a complex that degrades β-catenin
(see previous station…)

Classification of polyps?
• Non-neoplastic:
o Hamartomatous
o Metaplastic
o
P
s
e
u
d
o
p
o
l
y
p
s
a
s
i
n
U
C
• Neoplastic:
o Villous (40%)
o Tubulovillous (20%)
o Tubular (5%)

What does the malignant potential of adenomas depend on? Q: complications?


- Malignant transformation
• Type of adenoma >> Villous (40%) - Intestinal obstruction
• Diameter of adenoma: - Electrolytes disturbance (hypokalemia)
- Ulceration & Bleeding
o < 1 cm → 5% - Intsusseption
o 2 cm → 20 %
Q: Describe adenoma carcinoma sequence?
• Degree of dysplasia Stepwise accumulation of mutations of proto-oncogenes and
tumor suppressor genes:
Extracolonic manifestations in the related gardener syndrome? 1- Loss of APC (tumor suppressor gene) → hyperplasia
2- K-RAS (proto-oncogene) mutation → dysplasia
• Mandibular osteoma 3- Loss of p 53 (tumor suppressor gene) → adenocarcinoma
• Desmoid tumors
• Sebaceous cyst
• Gastric fundal polyps
Management?
Prophylactic near total colectomy by the age of 25

If the patient has a 2yo child, what is your advice?


Colonoscopic screening by the age of 12

Define endometriosis?
Endometriosis is defined by the presence of “ectopic” endometrial tissue at a site outside of the uterus
due to hormonal changes associated with epithelial changes.

Why do pt with endometriosis have pain?


- Due to release of substance p and adhesions.

41
PATHOLOGY MO’s MRCS B NOTES (Previously called Reda’s Notes) 41
Definitions
Dysplasia?
Disordered cellular development characterized by increased mitosis, pleomorphism without the ability to invade
the basement membrane
Severe dysplasia = carcinoma in situ
Abscess? Collection of pus surrounded by granulation or fibrous tissue
Pus? collection of neutrophils plus dead or dying micro organisms
Ulcer? a lesion in the mucous membrane or the skin resulting from the gradual disintegration of surface
epithelial cells
Secondary intention? secondary healing by reepithelization and contraction

Life style modifications to reduce risk of cancer colon?


• Fibre-rich diet
Reduce fat intake What is Staph. aureus ?
Gram +ve bacteria found in clusters
• Limit alcohol

• Stop smoking

STATION 4:

42
42 MO’s MRCS B NOTES (Previously Reda’s called Notes) PATHOLOGY
GB Cancer + Pseudomembranous Colitis

What is the commonest type of malignant neoplasm of the SCENARIO:


gall bladder? Cholecystectomy + surgical site infection
Adenocarcinoma Histology report describes malignant gall bladder
neoplasm
Causes of GB cancer?
• Cholelithiasis; gall stones
• Chronic cholecystitis

Risk factors? (Qs was about in UK)


• Age > 70 yrs
• Female sex
• Family history
• Ethnicity (Mexicans/native American)
• Smoking
• Gall stones (most common)
• GB polyp >1 cm
• Porcelain GB
• Chronic infection by S. typhus
• ABPJ
• Choledochal cyst
• Obesity

Where does GB cancer spread to first? 72 years old hair dresser admitted for elective cholecystectomy
following history of biliary colic she also have background of DM,
Direct invasion to liver (segment 4/5) HTN, hyperlipidemia and myxedema. At surgery she required
and local nodes
open resection for thickened and fixed GB with palpable stones,
Spread? the histopathology report show malignant GB neoplasm.
• Porta hepatis lymph nodes
• Liver (segment V)
• CBD
• Stomach
• Duodenum

Pathology?
Adenocarcinoma
3 days postoperatively patient abdominal wound breakdown with yellowish and smelly discharge noted, what happened?
SSI in POD3 with yellowish and smelly discharge, cause?
• Local sepsis, most likely to be bacterial # If you have no local cellulitis with minimal inflammation
• Actinobacillus (sulphur granules) around the wound, what you will do?
- open the wound
What is the most common organism causing surgical site infection? - take swap for culture and sensitivity
- clean the wound
Staph. aureus - allow drainage

Name three organisms likely to be associated with necrotizing fasciitis? (It a polymicrobial) give 3
• Group A streptococcus (Strep. Pyogenes)
• Staph. aureus
• Clostridium perfringens
• Bacteroides fragilis
Vibrio vulnificus

The pt condition deteriorates, now she has increasing pyrexia, tachycardia, reduced UOP, the wound site and local tissue
now has very intense congestion with purple and black discoloration with possible necrotic tissue, what happened?
- Necrotizing fasciitis

43
PATHOLOGY MO’s MRCS B NOTES (Previously called Reda’s Notes) 43
Diagnosis?
• Cellulitis
• Crepitus
• Laboratory: LRINEC (laboratory risk indicator for necrotizing fasciitis)
Score >= 6 → necrotizing fasciitis is highly considered
Ix Value Points
CRP ≥15 mg/dL (150 mg/L) 4
< 15 0
White blood cell count
15 - 25 1
(x10,000/µL)
> 25 2
Not asked > 13.5 0
Hemoglobin (g/dL) 11 – 13.5 1
< 11 2
Na+ (mEq/L) < 135 2
Creatinine > 141 µmol/L (>1.6 mg/dL) 2
Glucose > 10 mmol/L (>180 mg/dL) 1

Pathology?
Extensive necrosis with thrombosis of blood vessels

Management?
• Hemodynamic support according to CCrISP protocol
• Surgical debridement
• Antibiotics according to culture and sensitivity

Who to involve in care? After general supportive therapy, what is the two appropriate treatment for necrotizing fasciitis?
1. Surgical debridement of dead and infected tissues till viable and healthy areas of tissue
• ITU specialist
2. After taking swap for culture and sensitivity will start broad spectrum antibiotics according to
• Plastic surgeon hospital policy and after discussion with microbiologist.

Patient responds to treatment and is improving but 9 days later patient develops bloody diarrhea, cause? (Examiner want 4)
• Pseudomembranous colitis
• Ischemic colitis
• Hospital acquired infective gastroenteritis, norovirus
• Inflammatory bowel disease
Endoscopy done showing picture on previous page, describe what you see? Multiple yellowish plaques
Pathogenesis of PMC?
Pseudomembranous colitis is often triggered by antibiotic therapy that disrupts the normal microbiota and allows C.
difficile to colonize and grow. The organism releases toxins that disrupt epithelial function. The associated inflammatory
response includes characteristic volcano-like eruptions of neutrophils from colonic crypts that spread to form
mucopurulent pseudomembranes. Resulting in exudation of protein and inflammatory cells.

Q: how to suspect PMC?


Old, hospitalized, on long course of Abx developed diarrhea with abdominal pain and fever

44
44 MO’s MRCS B NOTES (Previously Reda’s called Notes) PATHOLOGY
STATION 5:
Diverticulitis + Endometriosis

SCENARIO:
Patient with LIF pain + peritonism → ruptured diverticulitis
Hartman's procedure was done
Histopathology revealed diverticulitis + endometriosis

Q: Pathophysiology of diverticulosis?
• Congenital
• Acquired (Constipation + Ageing)
Colonic diverticula result from the unique structure of the colonic muscularis propria and elevated intraluminal
pressure in the sigmoid colon. Where nerves, arterial vasa recta, and their connective tissue sheaths penetrate the
inner circular muscle coat, focal discontinuities in the muscle wall are created. In other parts of the intestine these
gaps are reinforced by the external longitudinal layer of the muscularis propria, but, in the colon, this muscle layer is
gathered into the three bands termed taeniae coli. Increased intraluminal pressure is probably due to exaggerated
peristaltic contractions, with spasmodic sequestration of bowel segments, and may be enhanced by diets low in fiber,
which reduce stool bulk, particularly in the sigmoid colon.

Complications?
• Infections
• Diverticulitis
Perforation, paracolic abscess, fecal peritonitis
• Fistula (colovesical, vaginocolic, ileocolic)

• Bleeding
• I.Obstruction

Cause of diverticulitis in diverticular disease?


Obstruction of diverticula leads to inflammatory changes, producing diverticulitis and peridiverticulitis. Because the wall
of the diverticulum is supported only by the muscularis mucosa and a thin layer of subserosal adipose tissue,
inflammation and increased pressure within an obstructed diverticulum can lead to perforation.

Classification for perforated diverticulitis?


Hinchey classification
I Pericolic abscess
IIa Distant abscess amenable to percutaneous drainage
IIb Complex abscess associated with fistula not amenable to percutaneous drainage
III Generalized purulent peritonitis
IV Fecal peritonitis

Management of diverticulitis? Terminology


Conservative (for uncomplicated diverticulitis) • Diverticulosis—the presence of
• Fluids diverticula that are asymptomatic
• NPO • Diverticular disease—diverticula
• Antibiotics associated with symptoms
• CT guided percutaneous drainage of collection should be considered • Diverticulitis—evidence of diverticular
inflammation (fever, tachycardia) with
Surgery (perforations): staged procedure:
or without localised symptoms and
• Diverting colostomy signs
• Hartmann • Complicated diverticulitis—
• Resection anastomoses perforation*, abscess, fistula,
stricture/obstruction
Investigations?
• FBC
• Urine
• CXR and AXR
• Rigid sigmoidoscopy
• Barium enema
• CT abdomen
Q: what is the layer between mucosa and muscularis? What its contents?
- Submucosa: contain connective tissue, blood vessels, lymphatic vessels, and submucosal nerve plexus of meissner)

Q: what are the contents of neutrophils?


- lysosomal enzymes, Azurophil, Specific, Gelatinase, and Secretory granules

45
PATHOLOGY MO’s MRCS B NOTES (Previously called Reda’s Notes) 45
How neutrophils migrate to the site of infection? (see diagram and text below)
(1) Margination and rolling along the vessel wall
(2) Firm adhesion to the endothelium
(3) Transmigration between endothelial cells
(4) Migration in interstitial tissues toward a chemoattractant stimulus

The multistep process of leukocyte migration through blood vessels, shown here for neutrophils. The leukocytes first
roll, then become activated and adhere to endothelium, then transmigrate across the endothelium, pierce the
basement membrane, and migrate toward chemoattractants emanating from the source of injury.
Different molecules play predominant roles in different steps of this process: selectins in rolling; chemokines (usually
displayed bound to proteoglycans) in activating the neutrophils to increase avidity of integrins; integrins in firm
adhesion; and CD31 (PECAM-1) in transmigration. ICAM-1, Intercellular adhesion molecule 1; PECAM-1 (CD31),
platelet endothelial cell adhesion molecule-1; TNF, tumor necrosis factor.

How did endometriosis get to the colon?


Theories are as follows:
• The regurgitation theory: proposes that
endometrial tissue implants at ectopic sites via
retrograde flow of menstrual endometrium.
Retrograde menstruation through the fallopian
tubes occurs regularly even in normal women
and can explain the distribution of
endometriosis within the peritoneal cavity.
• The benign metastases theory: holds that
endometrial tissue from the uterus can “spread”
to distant sites (e.g., bone, lung, and brain) via
blood vessels and lymphatic channels.
• The metaplastic theory: suggests that
endometrium arises directly from coelomic
epithelium (mesothelium of pelvis or abdomen),
from which the müllerian ducts and ultimately
the endometrium itself originate during
embryonic development. In addition,
mesonephric remnants may undergo
endometrial differentiation and give rise to
ectopic endometrial tissue. Pathogenesis of endometriosis
• The extrauterine stem/progenitor cell theory:
is a recent idea that proposes that
stem/progenitor cells from the bone marrow
differentiate into endometrial tissue.

46
46 MO’s MRCS B NOTES (Previously Reda’s called Notes) PATHOLOGY
Intraperitoneal picture?
Burn powder, dark blue, black, chocolate cysts

Describe epithelium of uterus?


Simple columnar supported by thick vascular stroma. The endometrium is the inner epithelial layer, along with its
mucous membrane. It has a basal layer and a functional layer; the functional layer thickens and then is sloughed during
the menstrual cycle.

Is endometriosis can increase the risk of cancer?


Yes, especially ovarian cancer (3.5-fold risk increase)

Why endometriosis causes pain?


Because of intrapelvic bleeding and periuterine adhesions and release of substance p

Few days later developed LIF collection why?


Retained collection (pelvic abcess)

What antibiotics to give? According to local policy and after discussion with microbiologist
• Amoxicillin clavulanic (1.2 g BD for 7 days) to cover gram +ve organisms
• Gentamycin (80 mg BD for 3 days) to cover gram -ve organisms
• Clindamycin (600 mg BD for 5 days) to cover anaerobes

STATION 6:

47
PATHOLOGY MO’s MRCS B NOTES (Previously called Reda’s Notes) 47
This page intentionally left blank

48
48 MO’s MRCS B NOTES (Previously Reda’s called Notes) PATHOLOGY
Peptic Ulcer Disease + Hyperparathyroidism
SCENARIO: Patient with gastric ulcer → hematemeses → OGD → peptic ulcer, biopsy was taken. Hypercalcemia
Define an ulcer?
An ulcer is a local defect of the mucous membrane or the skin due to gradual disintegration of the surface epithelial cells
OR
Breach of the continuity of skin epithelium or mucous membrane caused by sloughing out of inflamed necrotic tissue.

Risk factors of PUD? For this patient?


• H-pylori infection
• NSAIDs
• Smoking

What is H. pylori?
Gram negative microaerophilic spiral bacteria found in the stomach (Antrum)

CLO test (campylobacter like organism):


It depends on urease production by H. pylori
A gastric mucosal biopsy is taken during gastroscopy and is placed in a medium containing urea and an indicator such as
phenol red, urease production by H-pylori converts urea to ammonia which increase pH changing the colour (yellow to
red) → positive test

How H. pylori can survive in acidic medium?


H. pylori survives in acidic conditions by producing urease, which catalyzes hydrolysis of urea to yield ammonia thus
elevating the pH of its environment.

How does H. pylori cause gastritis?


H. pylori produce certain proteases and phospholipases and these together with toxic ammonia produced from
breakdown of urea, damage gastric mucosa and cause inflammation

Mechanism by which H-pylori can colonize the stomach?


1- Flagella, which allow the bacteria to be motile in viscous mucus
2- Urease, which generates ammonia from endogenous urea, thereby elevating local gastric pH around the
organisms and protecting the bacteria from the acidic pH of the stomach
3- Adhesins, which enhance bacterial adherence to surface foveolar cells
4- Toxins, such as that encoded by cytotoxin-associated gene A (CagA), that may be involved in ulcer or cancer
development by poorly defined mechanisms
These factors allow H. pylori to create an imbalance between gastroduodenal mucosal defenses and damaging forces that
overcome those defenses.

Type of gastric cancer that can be caused by H. pylori?


• Adenocarcinoma
• MALT (Mucosal associated lymphoid tissue tumor) Q: Which immune disease associated with it? Hashimoto thyroiditis

Eradication of H. pylori?
7 days twice daily of
Full dose of PPI + metronidazole 400 mg + clarithromycin 250mg, or
Full dose of PPI + amoxicillin 1g + clarithromycin 500mg

Mechanism of action of PPI


PPI binds irreversibly to H+/K+ ATPase enzyme (proton pump) on gastric parietal cells and blocks secretion of H+, which
combine with CI- in the stomach lumen to form HCL.
See ASSCC (Gastric Ulcer Station)

Action of HCL?
• Activated pepsinogen to pepsin which help in proteolysis
• Antimicrobial Q: What is normal gastric PH?
• Provide optimal PH for action of pepsin 2
• Stimulating small intestine to release secretin and CCK. Q: If PH is 5 will you insert NG tube?
• Enhance absorption of Ca and Ferus Ion in small intestine. No

49
PATHOLOGY MO’s MRCS B NOTES (Previously called Reda’s Notes) 49
How can NSAIDs causes PUD?
• Topical irritant effect on the epithelium
• Impairment of the barrier properties of the mucosa
• Suppression of gastric PG synthesis (inhibitors of cyclooxygenase) (COX 1)
• Reduction of gastric mucosal blood flow
• Interference with the repair of superficial injury

Other causes of hematemesis in this patient?


Hypercalcemia → increased gastrin release → increased HCl production –> Mucosal erosion –> bleeding

Common causes of hypercalcemia?


• Malignancy
• Hyperparathyroidism (PTH adenoma)
• Renal failure

Cause of UTI in this patient? Q:What is the most common cause of primary hyperpathyroidism?
Renal stones - solitary parathyroid adenoma

How to localise parathyroid glands?


• Sestamibi scan (pre-operative)
• Frozen section (intra-operative)

What is frozen section?


It’s a pathological laboratory procedure to perform rapid microscopic analysis of a specimen.

How the specimen is fixed? (No details)


The surgical specimen is placed on a metal tissue disc which is then secured in a chuck and frozen rapidly to about -20 to
-30 °C. The specimen is embedded in a gel like medium called OCT and consisting of poly ethylene glycol and polyvinyl
alcohol. Subsequently, it is cut frozen with the microtome portion of the cryostat, the section is picked up on a glass slide
and stained (usually with hematoxylin and eosin, the H&E stain).

Why we cannot use paraffin based histopathology intraoperatively?


As it takes a week for paraffin to embed through the tissues

Epithelium of gastric mucosa in antrum?


Simple columnar with mucosal and goblet cells

What immune endocrine diseases and malignancy associated with helicobacter infections?
• Hashimoto's thyroiditis
• MALT

How many platelets present in packed RBC’s?


No platelets in packed RBC’s

After removing 4 parathyroid glands…


You are given the following histopathology report:
1 gland 0.2 g chief cells
3 glands ranging from 0.08 to 0.09 g oxyphilic cells and fat cells
What’s your interpretation?
Parathyroid adenoma in one gland with involution to the other glands

Histology of parathyroid adenoma?


• Uniform, polygonal chief cells with small, centrally placed nuclei.
• A few nests of larger oxyphil cells are present as well. Uncommonly, adenomas are composed entirely of this
cell type (oxyphil adenomas).
• A rim of compressed, non-neoplastic parathyroid tissue, generally separated by a fibrous capsule, is often visible
at the edge of the adenoma

Microscopic picture of parathyroid hyperplasia?


See MEN station

50
50 MO’s MRCS B NOTES (Previously Reda’s called Notes) PATHOLOGY
Where to find parathyroid gland if you do not see them in the normal position?
The superior mediastinum
(As the thymus originates from the third branchial pouch , it occasionally drags the inferior glands down to
the mediastinum)

Treatment of parathyroid adenoma?


Excision

Types of hyperparathyroidism? And cause?


Disease type Hormone profile Clinical features Cause
Primary • PTH (↑) • May be asymptomatic if mild Most cases due to solitary
hyperparathyroidism • Ca2+ (↑) • Recurrent abdominal pain adenoma (80%), multifocal
• Phosphate (↓) (pancreatitis, renal colic) disease occurs in 10-15% and
• Urine Ca2+: creatinine • Changes to emotional or parathyroid carcinoma in 1%
clearance ratio > 0.01 cognitive state or less
Secondary • PTH (↑) • May have few symptoms Parathyroid gland hyperplasia
hyperparathyroidism • Ca2+ (↓ or normal) • Eventually may develop bone occurs as a result of low
• Phosphate (↑) disease, osteitis fibrosa cystica calcium, almost always in a
• Vitamin D levels (↓) and soft tissue calcifications setting of chronic renal failure
Tertiary • PTH (↑) • Metastatic calcification Occurs as a result of ongoing
hyperparathyroidism • Ca2+ (Normal or ↑) • Bone pain and / or fracture hyperplasia of the parathyroid
• Phosphate levels (↓ or • Nephrolithiasis glands after correction of
Normal) • Pancreatitis underlying renal disorder,
• Vitamin D (Normal or ↓) hyperplasia of all 4 glands is
• ALP (↑) usually the cause

Treatment of hypercalcemia?
• Hydration
• Forced diuresis
• Bisphosphonates: IV pamidronate
• Calcitonin

51
PATHOLOGY MO’s MRCS B NOTES (Previously called Reda’s Notes) 51
STATION 6:
Esophageal Carcinoma

What is the normal esophagus lining? SCENARIO:


Non keratinized stratified squamous epithelium Patient, smoker with dysphagia
Hx of GERD and loss of weight
What is the most probable diagnosis? (20 pounds)
Cancer esophagus

What the etiological factors of cancer esophagus in this patient?


• Chronic GERD
• Smoker
• Alcoholic
What is the effect of prolonged GERD?
Chronic GERD leads to the development of Barret’s esophagus with increased risk of developing adenocarcinoma

Which test to do for this patient?


Endoscopy plus EUS with FNAC and will take biopsy

What is the obtained cells will tell you?


• Stage
• Grade
• Immunohistochemistry

What is the stain used in immunohistochemistry? Treatment options of esophageal cancer?


According to stage
Cytokeratin If operable >> feeding jejunostomy + esophagectomy (iver lewis or MC-queen) + chemoradiotherapy
If non-operable >> debulking + Self expanding metallic stent + palliative chemoradiotherapy

Pathology report showed Barret’s esophagus Columnar metaplasia of the stratified squamous epithelium of the
esophagus Type of lining: columnar epithelium Type of cancer: adenocarcinoma. Staging: TNM [T2, N1]

51
PATHOLOGY MO’s MRCS B NOTES (Previously called Reda’s Notes) 51
T status
Tis High-grade dysplasia
T1 Invasion into the lamina
propria, muscularis mucosae,
or submucosa
T2 Invasion into muscularis
propria
T3 Invasion into adventitia
T4a Invades resectable adjacent
structures (pleura,
pericardium, diaphragm)
T4b Invades unresectable
adjacent structures (aorta,
vertebral body, trachea)
N status
N0 No regional lymph node
metastases
N1 1 to 2 positive regional lymph
nodes
N2 3 to 6 positive regional lymph
nodes
N3 7 or more positive regional
If patient develops pleural effusion, what are the possible causes? lymph nodes
• Spread of cancer cells to pleura M status
• Lung metastases M0 No distant metastases
• Obstruction of the thoracic duct M1 Distant metastases
Histologic grade
What pathological test to do? G1 Well differentiated
Effusion cytology G2 Moderately differentiated
G3 Poorly differentiated
How to treat? G4 Undifferentiated
Palliative treatments
• Thoracentesis,
• Indwelling pleural catheters (IPCs),
• Pleurodesis,
• Pleuroperitoneal shunting (PPS).

Developed enlargement of a supraclavicular lymph node, what test/investigation to do?


FNAC Or excisional biopsy

Features under the microscope: metastatic adenocarcinoma with tumor cells having hyperchromatic eccentric nuclei and
intracytoplasmic vacuolation

Investigations of MI?
• ECG
• Troponin
• CK-MB

What is troponin?
A complex of three regulatory proteins (troponin C, troponin I, and troponin T) that is integral to muscle contraction

Where is it found?
Cardiac muscle (myocardium) and skeletal muscle

52
52 MO’s MRCS B NOTES (Previously Reda’s called Notes) PATHOLOGY
STATION 10:
Gastric Carcinoma

On entering you are given a histopath report showing SCENARIO:


o Signet ring carcinoma Patient with LIF pain + gastric cancer + OA
o Positive LN Surgery was done (gastrectomy with splenectomy)
o Margin Positive Pathology report inside
o Spleen involved

4 2 major risk factors for gastric cancer? WHO classification


• H. pylori infection and family history for gastric cancer. • Papillary adenocarcinoma
• Chronic atrophic gastritis • Tubular adenocarcinoma
• Intestinal metaplasia • Mucinous adenocarcinoma
• Pernicious anemia • Signet-ring cell carcinoma and other poorly
• Adenomatous polyps more than 2 cm cohesive carcinoma
• Previous gastric surgery • Mixed carcinoma
• HNPCC o Adenosquamous carcinoma
• FAP o Squamous cell carcinoma
o Hepatoid adenocarcinoma
Pathogenesis? o Carcinoma with lymphoid stroma
Gastric cancer is believed to develop by a sequence of o Choriocarcinoma
pathological changes o Carcinosarcoma
Normal mucosa → Chronic gastritis → Intestinal metaplasia → o Parietal cell carcinoma
Dysplasia → Intramucosal carcinoma → Invasive gastric o Malignant rhabdoid tumor
carcinoma o Mucoepidermoid carcinoma
o Paneth cell carcinoma
Discuss pathology report with the family in 4 simple lines? o Undifferentiated carcinoma
• This is cancer of the stomach o Mixed adeno-neuroendocrine carcinoma
• With incomplete resection (Not completely removed) o Endodermal sinus tumor
• With high possibility of recurrence (It may come back again) o Embryonal carcinoma
• The patient will require further resection and o Pure gastric yolk sac tumor
chemotherapy o Oncocytic adenocarcinoma

What is the commonest histological type of gastric cancer?


Adenocarcinoma 95%
Q: Other types? GIST, Hereditary defused gastric cancer, MALToma, Carcinoid, leomayosarcoma, lymphoma
Classification?
The WHO classification: 5 main types
• Tubular adenocarcinoma – This is made up of different sized small branching tubes (tubules).
• Papillary adenocarcinoma – This tumor grows outward from the stomach wall and contains finger-like
growths that stick out into the stomach cavity. The cells tend to look and behave much like normal cells.
• Mucinous adenocarcinoma – There is a lot of mucin (the main substance in mucus) outside of the cancer
cells.
• Poorly cohesive carcinomas (including signet ring cell carcinoma and others) – These are arranged into
clumps of cancerous cells.
• Mixed carcinoma – This contains a mix of types of adenocarcinomas of the stomach.
The classification is based on the predominant histologic pattern of the carcinoma which often co-exists with less
dominant elements of other histologic patterns. Tubular adenocarcinoma is the most common histologic type of
early gastric carcinoma
‫لورين ابو اتنني‬
The Lauren classification divides adenocarcinoma of the stomach into 2 main types:
• Intestinal type – Tumor cells are well differentiated, grow slowly and tend to form glands. This type is found
more often in men than in women and occurs more often in older people.
• Diffuse type – tumor cells are poorly differentiated, behave aggressively and tend to scatter throughout the
stomach (rather than form glands). This type spreads to other parts of the body (metastasizes) much
quicker than intestinal type tumors. The diffuse type occurs equally among men and women and tends to
develop at a younger age than the intestinal type.

Q: After splenectomy what is your plan for the patient?


- Give vaccine for the patient after 2 weeks against polysaccharide encapsulated organisms (N. meningitides, H.Influanzae, Sterp. Pneumoniae)
- Prophylactic antibiotics to protect the patient against post-splenectomy overwhelming infection (OPSI)

53
PATHOLOGY MO’s MRCS B NOTES (Previously called Reda’s Notes) 53
Q: How gastric cancer spread first? By lymphatics to LN (left gastric LN)
Q: Other routes? Hematogenous and Trans-celomic

7-10 days later the patient had axillary vein thrombosis, what predisposes to that?
• Hypercoagulable state in malignancy
• Venous stasis from Virchow LN

6 months later came with ascites, deranged liver functions, hepatic metastasis Mention 2 pathological tests to do?
• Ascites tap and cytology
• Liver biopsy from metastasis
• FNAC from the left supraclavicular lymph node
• Tumor marker: CA72-4 (For recurrence)

Treatment of this patient?


• Feeding jejunostomy
• Palliation of ascites by repeated tapping
• Pain relief using opioids
• Palliative chemotherapy

Mechanism of malignant ascites in cirrhotic liver?


•LowOncotic pressure is high (Hypoalbuminemia)
• Interstitial pressure is increased(Portal Hydrostatic pressure)
See ASSCC (Esophageal Varices and Hematemesis station)

Borrmann classification system for gastric cancer?


Macroscopic appearance of the lesion:
• Polypoid growth
• Fungating growth
• Ulcerating growth
• Diffusely infiltrating growth (linitis plastica)
Define para-neoplastic syndrome?
Which para-neoplastic conditions associated with gastric cancer?
• Acanthosis nigricans
• Dermatomyositis.

What are the procedure specific complications of total gastrectomy?


Early
• Anastomotic leak
• Pancreatitis
• Cholecystitis
• Hemorrhage
• Infection.
Late
• Dumping syndrome
• Vitamin B12 deficiency (lack of intrinsic factor)
• Metabolic bone disease
• Recurrence of malignancy

What is dumping syndrome?


Loss of the reservoir function of the stomach (e.g. following gastrectomy) results in the rapid transit of highly osmotically
active substances into the duodenum following meals and may cause ‘dumping syndrome’.
Early dumping: 30–60 minutes following a meal, rapid transit of the hyperosmolar gastric contents into the small bowel
results in a fluid shift from the intravascular compartment to the gastric lumen and small bowel distension, colicky
abdominal pain, diarrhoea and vasomotor symptoms, such as tachycardia and postural hypotension.
Late dumping: 1–3 hours following meals. Rapid transit of carbohydrate into the small bowel results in sudden
absorption of high levels of glucose and compensatory hyperinsulinemia, resulting in subsequent hypoglycaemia.

54
54 MO’s MRCS B NOTES (Previously Reda’s called Notes) PATHOLOGY
STATION 7:
Cancer Colon + Hemorrhoids + Myocardial Infarction

SCENARIO: PR bleeding diagnosed as hemorrhoids, colonoscopy done revealed adenocarcinoma with melanosis coli,
biopsied, histopathology revealed lesion up to muscularis layer

Dukes’ staging?

Lung
Liver
Bone
Skin

What is hemorrhoids?
Swollen or inflamed anal cushions
• Swollen and bulging veins in the lower rectum and anus
Pathogenesis of hemorrhoids?
• The anal cushions function normally when they are fixed to their proper sites within the anal canal by
fibromuscular ligaments, which are the anal remnants of the longitudinal layer of the muscularis propria from
the rectum (Treitz's ligaments).
• When these submucosal fibres fragment (as by prolonged and repeated downward stress related to straining
during defecation), the anal cushions are no longer restrained from engorging excessively with blood and may
result in bleeding and prolapse.

Pathogenesis of thrombosed hemorrhoids?


• Acute thrombosis results from sudden raised pressure causing rupture of the vascular plexus leading to a blood
clot at the anal verge.
• Any activity requiring excessive abdominal straining such as over-exercising can cause this

Why coronary arteries get thrombosed?


Due to coronary artery atherosclerosis

Define melanosis coli?


It is a disorder of pigmentation of the wall of the colon, often identified at the time of colonoscopy. It is benign and may
have no significant correlation with disease. Associated with chronic laxative use.

Define adenoma?
The term adenoma is generally applied to benign epithelial neoplasms producing gland patterns and to neoplasms
derived from glands but not necessarily exhibiting glandular patterns.

55
PATHOLOGY MO’s MRCS B NOTES (Previously called Reda’s Notes) 55
Endothelial injury

How coronary arteries get atherosclerosis?


• Endothelial injury—and resultant endothelial
dysfunction—leading to increased
permeability, leukocyte adhesion, and
thrombosis
• Accumulation of lipoproteins (mainly oxidized
LDL and cholesterol crystals) in the vessel
wall
• Platelet adhesion
• Monocyte adhesion to the endothelium,
migration into the intima, and differentiation
into macrophages and foam cells
• Lipid accumulation within macrophages,
which release inflammatory cytokines
• Smooth muscle cell recruitment due to
factors released from activated platelets,
macrophages, and vascular wall cells
• Smooth muscle cell proliferation and ECM
production

Risk factors of atherosclerosis?


Non-modifiable (Constitutional)
• Genetic abnormalities
• Family History
• Increasing age
• Male gender
Modifiable
• Hyperlipidemia
• Hypertension
• Cigarette smoking
• Diabetes
• Inflammation

What is the function of thrombus?


A thrombus is a healthy response to injury intended to
prevent bleeding

How it is formed?
The following sequence of events takes place:
• An atheromatous plaque is suddenly
disrupted by intra-plaque hemorrhage or
mechanical forces, exposing sub-endothelial
collagen and necrotic plaque contents to the
blood.
• Platelets adhere, aggregate, and are
activated, releasing thromboxane A2,
adenosine diphosphate (ADP), and
serotonin—causing further platelet
aggregation and vaso-spasm.
• Activation of coagulation by exposure of
tissue factor and other mechanisms adds to
the growing thrombus.
• Within minutes, the thrombus can evolve to
completely occlude the coronary artery
lumen.
Response to injury in atherogenesis: 1, Normal. 2, Endothelial injury
with monocyte and platelet adhesion. 3, Monocyte and smooth muscle
cell migration into the intima, with macrophage activation. 4,
Macrophage and smooth muscle cell uptake of modified lipids and
further activation. 5, Intimal smooth muscle cell proliferation with ECM
elaboration, forming a well-developed plaque.

56
56 MO’s MRCS B NOTES (Previously Reda’s called Notes) PATHOLOGY
The natural history, morphologic features, main pathogenic events, and clinical complications of atherosclerosis

Mechanism of myocardial infarction?


• Within seconds of vascular obstruction, aerobic glycolysis ceases, leading to a drop in adenosine triphosphate
(ATP) and accumulation of potentially noxious metabolites (e.g., lactic acid) in the cardiac myocytes
• The functional consequence is a rapid loss of contractility, which occurs within a minute or so of the onset of
ischemia. Ultrastructural changes (including myofibrillar relaxation, glycogen depletion, cell and mitochondrial
swelling) also become rapidly apparent.
• These early changes are potentially reversible. Only severe ischemia lasting at least 20 to 40 minutes causes
irreversible damage and myocyte death leading to coagulation necrosis

57
PATHOLOGY MO’s MRCS B NOTES (Previously called Reda’s Notes) 57
STATION 8:
STATION 11:
This page intentionally left blank

58
58 MO’s MRCS B NOTES (Previously Reda’s called Notes) PATHOLOGY
Malignant Melanoma

Definition? SCENARIO:
Malignant neoplasm of melanocytes (melanin producing cells), mainly Skin lesion, you are given a pathology report
arising in skin.
Other sites: nasal cavities, retina, gastrointestinal mucosa
Q7. Name 2 inherited condition with increased risk of malignant melanoma?
Skin conditions associated with melanoma? 33 years old man referred to outpatient clinic by his GP following a skin
biopsy of forearm lesion. His report reads: skin biopsy shows epidermis
• Xeroderma pigmentosa and dermis in which there is proliferation of atypical non-pigmented
• Albinism epithelioud melanocytes at the junction and invading the dermis up to
the deep margin, the features of those malignant melanoma.
• Giant congenital pigmented naevus
• Fitzpatrick skin type 1
• Dysplastic naevus syndrome.

Other risk factors of malignant melanoma?


• Hutchinson’s melanotic freckles
• Immunocompromised patients
• Past history of melanoma
• Red hair, sun exposure

Comment on the pathology report?


• Size
• Breslow thickness: measured in mm. From the top of stratum granulosum to the deepest point of tumour
involvement
• Depth of invasion (Clark’s levels)
• Ulceration, mitoses, lymphovascular invasion
• Immunohistochemistry staining

Q1.
Melanoma SCC
W
h
a
t
i
s
t
h
e
b
a
s
i
c
d
i
f
f
e
r
e
n
c
e
b
e
t
w
e
e
n
m
e
l
a
n
o
m
a
a
n
d
S
C
C
?
Answer only: Arise from the lower layer of the epidermis from any Arise from upper and mid layer of epidermis with keratin
part of the body and its tumor of Melanocytes. pearl formation usually on sun exposed areas arise from epithelial cells
Needs intermittent sun exposure Nonmelanoma need continuous sun exposure
Arise from lower most layers of epidermis Arise from superficial layers
Develops in younger people Older people
Anywhere in body Sun exposed areas
Appears as red bump, scaly patch or non-healing sore Asymmetrical mole with irregular border and multicolored
Incidence more common Less common
Metastasis less common More common
Better prognosis Poorer prognosis
‫ حاجات معكوسة‬4 ‫اخر‬

Q2. What treatment is indicated as consequence of pathology report?


- Do further surgery to achieve complete resection.

Q5. What pathological features can be regarded as good prognostic features from histology report?
- limited tumor depth of invasion.
- No lymphocytic infiltration.

Q6. Why sun exposure is a risk factor for malignant melanoma?


- high energy radiation in the ultraviolet can damage DNA causing mutations.

59
PATHOLOGY MO’s MRCS B NOTES (Previously called Reda’s Notes) 59
Difference between BCC and SCC?
Basal cell carcinoma Squamous cell carcinoma
Spreads less commonly Spreads more commonly than BCCs
Found deep within skin layers of thick skin Found near body orifices and in the oral cavity
Occupies basement membrane Occupies outer layer of skin
Most common form of NMSC Less common but more dangerous
96% of victims 40 years or more Almost all victims are 40 years or older
Red, pale, or pearly in color Thickened, red, spot, may bleed, crust or ulcerate

Q3. Describe 3 histopathological features of invading melanoma that indicate an aggressive potential of melanoma and likely poor prognosis?
Poor prognostic factors?
• Increased tumor thickness (Breslow thickness)
• Increased depth of invasion Deep dermal or subcutecular tissue invasion.
• Type of melanoma: nodular, amelanotic melanoma
• Presence of ulceration
• Presence of lymphatic or perineural invasion or blood vessel invasion.
• Presence of regional or distant metastasis
• Male, old age

Genes responsible for familial MM? BRAF gene


CDKN2A and CDK4, MC1R, BRCA2
Q8. The pt undergo a complete excision of 1 mm thick melanoma with clear margins about 0.5 mm, what is likely next step in treatment?
Lesion excised, Breslow thickness 1.5 mm, margins 0.5 mm, what to do?
Re-excision
- Wider excision with additional boundaries to be determined by Breslow thickness of melanoma.

Management?
Excision + Safety margin
Size of the safety margin is based on the Breslow thickness
• <0.76mm thickness= 1 cm margin
• 0.76-1mm thickness= 2 cm margin
• 1mm= 3 cm margin
Q9. What intra-operative investigation can assist you to provide a comment that the margin is clear?
What to do to ensure adequate margins?
• Frozen section
• Mohs microsurgery (Mohs micrographic surgery)

Post excision the patient developed regional lymphadenopathy, management?


• FNAC
• Whole body CT

60
60 MO’s MRCS B NOTES (Previously Reda’s called Notes) PATHOLOGY
of axillary tissues
and distal edema
Q10. Post excision the patient developed painful swelling of the arm + dyspnea, cause?
• Axillary vein thrombosis → pulmonary embolism

How to treat DVT? (Not here)


• Acute treatment with parenteral anticoagulation (LMWH, fondaparinux).
• Maintaining patients on anticoagulation for at least 6 months is the standard of practice.
• Warfarin, to keep INR 2-3, standard doses range between 1–10 mg per day for 6 months
• Catheter-directed thrombolysis (CDTL) if clot is less than 14 days in duration or acute phlegmasia cerulea
dolens inpatients with no contraindications to thrombolytic therapy. A clot present for more than 14 days
leads to thrombus organization that limits the effectiveness of thrombolysis. Use tPA continuous infusion of
0.5–1 mg per hour for at least 8 hours (an initial bolus, every 6–8 hours monitor fibrinogen levels, which
should be kept above 100 mg/dL to avoid depletion. Fibrinogen levels below 100 mg/dL can cause
hemorrhagic complication. Upon termination of the procedure, the patients are systemically anticoagulated
with warfarin for 6 month
• Indications for SVC filter placement are failure or contraindication to therapeutic anticoagulation or for
presurgical prophylaxis in the setting of substantial thromboembolic risk factors

Q11. Risk factors for thrombosis? in this patent (give me 2) ?


Virchow's triad:
• Hypercoagulable state
• Venous stasis
• Endothelial injury

Management (suspecting PE)?


• CTPA
• SC LMWH if the patient is hemodynamically
stable
• Thrombolysis or embolectomy (massive
embolism)

Malignant Melanoma Metastasis

DD? SCENARIO: A patient known to have


Inguinal lymph nodes receive lymphatic drainage from the lower hypothyroidism with a hard swelling in
extremities and skin of the lower abdomen, genitals, and perineum. her right inguinal region, her GP sent
Infections associated with inguinal lymph node: her for biopsy
• Cellulitis of the lower extremities
• Venereal infections - Syphilis, chancroid, herpes simplex virus
infection, lymphogranuloma venereum
Malignancies associated with inguinal lymph nodes:
• Lymphomas
• Metastatic melanomas from lower extremity primary site
• Squamous cell carcinomas from genital primary site

Q12. Results of histopath. Revealed malignant melanoma, what are types of malignant melanoma? (Examiner: Give me 4 types)
Melanomas are divided into 5 main types, depending on their location, shape and whether they grow outward or
downward into the dermis:
• Lentigo maligna: usually occur on the faces of elderly people
• Superficial spreading or flat melanoma: grows outwards at first to form an irregular pattern on the skin with an
uneven color (Most common 70%)
• Desmoplastic melanoma: is a rare malignant melanoma marked by non-pigmented lesions on sun-exposed
areas of the body
• Acral melanoma: occurs on the palms of the hand, soles of the feet, or nail beds
• Nodular melanomas: are lumpy and often blue-black in color and may grow faster and spread downwards (Bad prognosis)

Q13. What is Epitheloid Melanoma?


Melanoma cells can be categorized into 2 major types: epithelioid and spindle cells.
Epithelioid cells are large and round with abundant eosinophilic cytoplasm, prominent vesicular nuclei and large nucleoli.
They most commonly arise in superficial spreading and nodular melanomas.
Type of melanoma with cells resemble epithelium because of abundant eosinophilic cytoplasm and enlarged round to oval shaped nuclei.

61
PATHOLOGY MO’s MRCS B NOTES (Previously called Reda’s Notes) 61
Q4. What is a satellite lesion?
It is a form of local spread of malignant melanoma by contiguity and continuity leading to pigment spreading to the
surrounding area. They are found within 2 cm of the primary tumor.

Q14. How can you know the phenotype of the tumor?


Immunohistochemistry

Recent histopathological studies of lymph node in metastatic melanoma other than IHC?
Genetic mutation studies (BRAF gene)

Genetic subtypes
These molecular subtypes are based on the distinct genetic changes of the melanoma cells (mutations).
These genetic changes include:
Ask about this gene •
only BRAF mutations: The most common genetic change in melanoma is found in the BRAF gene, which is mutated in
about 50% of cutaneous melanomas.
• NRAS mutations: NRAS is mutated in the tumors of around 20% of patients with melanoma
• NF-1 mutations: NF-1 mutations are present in the tumors of around 10% to 15% of patients with melanoma
• KIT mutations: These mutations occur more commonly I melanomas that develop from mucus membranes,
melanomas o the hands or feet, or melanomas that occur in chronically sun-damaged skin, such as lentigo
maligna melanoma.

Q15. Where would you examine this lady?


• Primary sites: whole lower limb including nail beds and soles
• Metastatic sites: chest, abdomen and brain

Q16. How would you to treat this lady?


Excision of the primary lesion with safety margin plus block inguinal lymph node dissection plus radiotherapy

Q17. Post-operative the wound is red and swollen, culture was done revealed diplococci
Examples of gram-negative diplococci?
• Neisseria sp.
• Moraxella catarrhalis
• Acinetobacter
• Haemophilus
• Brucella
Examples of gram-positive diplococci?
• Streptococcus pneumoniae
• Enterococcus.

Q18. Next the patient got toxemia with rapidly spreading infection, what do you think?
Necrotizing fasciitis

Q19. What is SIRS?


Systemic Inflammatory response syndrome
Manifestations of SIRS include, two or more of the following:
• Body temperature less than 36 °C (96.8 °F) or greater than 38 °C (100.4 °F)
• Heart rate greater than 90 bpm
• Tachypnea (high respiratory rate), with greater than 20 breaths per minute; or, an arterial partial pressure of
carbon dioxide less than 4.3 kPa (32 mmHg)
• White blood cell count less than 4000 cells/mm³ (4 x 109 cells/L) or greater than 12,000 cells/mm³ (12 x 109
cells/L); or the presence of greater than 10% immature neutrophils (band forms). Band forms greater than 3% is
called bandemia or a "left-shift."
• Hyperglycemia (blood glucose > 6.66 mmol/L i.e. 120 mg/dL) in absence of diabetes mellitus
• Altered mental state

Q20. What happens to lung in SIRS?


ARDS
Define ARDS? See Burn / ARDS station in ASSCC
Diffuse alveolar damage and lung capillary endothelial injury

62
62 MO’s MRCS B NOTES (Previously Reda’s called Notes) PATHOLOGY
STATION 14:

First stem:
Second stem:
Basal Cell Carcinoma (BCC)

Describe the lesion?


SCENARIO: Lump over arm. Excision biopsy done,
Pearly papule with a central ulcer, with granulation tissue on base shown report BCC with depth of invasion, deep
and with rolled in (inverted) edges with surrounding margin involvement
telangiectasia

Why the surrounding skin is red?


1. Due to the presence of dilated subepidermal blood vessels.(New)
2. Inflammation.

Most probable diagnosis?


BCC

Differential Diagnosis?
• BCC
• SCC
• TB
• Actinic keratosis
• Seborrheic keratosis
• Verruca vulgaris

Natural Hx of BCC?
• Indolent with slow progression
• Locally destructive but limited
potential to metastasize (never
metastasize)

How lymph node spread occurs?


• Permeation
• Embolization
Tumor cell secret growth factors that promote lymph
angiogenesis in surrounding and the draining LN
What is your concern in pathology report?
Deep margin involvement, Invasion, size

How would you manage a pt with deep margin involvement?


Re-excision with safety margins, with frozen section or Mohs microsurgery

Treatment options for basal cell carcinoma?


• Surgical
o Curettage and Electrodessication: scraping away the tumor and stopping bleeding with cautery
o Excision with primary closure: flaps, grafts, and secondary intention healing. Excision margin of 4 mm
around the tumor is recommended where possible
o Cryotherapy (with liquid nitrogen), but can't obtain tissue biopsy
o Mohs micrographic surgery – Serial tangential horizontal sections are taken and examined histologically
until all margins are clear
• Radiotherapy
• Topical
o Topical photodynamic therapy - δ-aminolevulinic acid made up in a 20% emulsion and applied topically.
Tumor tissue absorbing this porphyrin metabolite becomes photosensitive with its conversion to
protoporphyrin IX and subject to photo destruction when exposed to light, usually in the wavelength
range 620-670 nm
o Topical fluorouracil 5%
o Topical imiquimod 5%

How to prevent recurrence of deep margin involvement during re-operation?


If recurrent, go for Mohs micrographic surgery (frozen section)

Skin graft placed for pt and subsequently had graft failure. Cause for graft failure?
Wound infection and Hematoma

63
PATHOLOGY MO’s MRCS B NOTES (Previously called Reda’s Notes) 63
Common organism?
S. aureus.& Streptococcus
What will do?
C&S and antibiotics according to hospital policy after discussing with microbiologist
Wound C&S grew MRSA, what is MRSA?
Methicillin-resistant Staphylococcus aureus.
# giving you a report show E. coli infection in culture. What is significance? - Contamination
- Pt immuno-compromised
How would you manage this pt with MRSA wound infection?
- First isolation of patient then inform infection control team and call microbiologist for advice
If abscess, I&D
Outpatient
• Oral Antibiotics as clindamycin, amoxicillin plus tetracycline or tmp/smx, linezolid*
Inpatient: Parenteral
• Vancomycin Dose to target trough level 7-14days
• Linezolid 600 mg twice daily, PO or IV 7-14
• Daptomycin 4 mg/kg once daily 7-14
• Telavancin 10 mg/kg once daily 7-14
• Clindamycin 600 mg IV or 300 mg PO 3times daily
Decolonization with mupirocin nasal or chlorhexidine for body decolonization

Infection control of MRSA: (No details needed)


Measures apply to all patients, regardless of MRSA status:
• High standards of hand decontamination are required to minimize the risk of cross infection
• Hands should be decontaminated before and after every patient contact
• Handwashing should be with liquid soap and water
• Alcohol hand rub may be used as an alternative
• Maintain high standards of aseptic technique
• Maintain high standards of ward cleanliness
• All linen should be handled in accordance with Trust Laundry Policy
• All waste should be disposed of in accordance with Trust Policy
• Re-usable equipment must be decontaminated before use on another patient.
Trust
• Decontamination and Disinfection Guidelines should be followed
• Antibiotics should be used in accordance with Trust Antibiotic Guidelines
• Minimize inter-ward transfer of patients
• Avoid overcrowding of patients
• Maintain adequate and appropriately skilled nursing and other staff levels
The following additional precautions should be used for all MRSA positive patients:
• Place patient in a single room
• If a side room is unavailable, may be managed in ward bays next to a sink following agreement
with the Infection Control Team
• If a number of MRSA positive patients are present, these may be managed in a cohort. This should
only occur following discussion with the Infection Control Team
• Wear gloves and disposable plastic aprons when handling the patient or having contact with their
immediate environment
• All waste should be regarded as clinical waste and to be disposed of in yellow waste bags
• All linen to be treated as contaminated/infected and to be disposed of in an inner red alginate bag
or alginate seamed/stitched bag placed within a white plastic outer
• Gowns may be required where extensive contact with the patient is anticipated

After excision, the patient developed regional lymphadenopathy? FNAC was done and revealed (lymphocytes, PMNL,
histiocytes, cells with an bilobed nuclei) Interpret?
Reed-Sternberg cells: (owl eye appearance) → Hodgkin’s lymphoma

Mechanism of lymphatic spread?


Malignant tumors release growth factors such as VEGF-C to induce lymphatic vessel expansion (lymph angiogenesis) in
primary tumors and in draining sentinel LNs, thereby promoting LN metastasis

64
64 MO’s MRCS B NOTES (Previously Reda’s called Notes) PATHOLOGY
STATION 5:
Sickle Cell Disease + Brain Tumor (AR)

What is sickle cell disease? SCENARIO:


Sickle cell disease is a common hereditary hemoglobinopathy caused by a point A lady known to have sickle
mutation in β-globin that promotes the polymerization of deoxygenated cell disease, had head injury.
hemoglobin, leading to red cell distortion, hemolytic anemia, microvascular CT was done and a temporal
obstruction, and ischemic tissue damage. mass of 3.8 cm was
HbS is produced by the substitution of valine for glutamic acid at the sixth amino acid accidentally discovered
residue of β-globin. In homozygotes, all HbA is replaced by HbS, whereas in
heterozygotes, only about half is replaced.
Q: Mood of inheritance?
Autosomal recessive
Complications?
• Vaso-occlusive crisis: triggered by infection,
dehydration, acidosis, affecting the bones
(painful bone crisis as in hand-foot
syndrome), lungs (acute chest syndrome),
brain (stroke and retinopathy), spleen
(autosplenectomy)
• Sequestration crisis: in children, massive
entrapment of sickle cell in the spleen will
lead to rapid splenic enlargement and
hypovolemic shock
• Aplastic crisis: due to infection of red cell
progenitors by parvo-virus
• Chronic tissue hypoxia: organ damage
(spleen, heart, kidney, lungs)
• Increased susceptibility of infection with
encapsulated organisms

Mechanism of autosplenectomy?
In early childhood, the spleen is enlarged up to
500 gm by red pulp congestion, which is caused by
the trapping of sickled red cells in the cords and
sinuses. With time, however, the chronic
erythrostasis leads to splenic infarction, fibrosis,
and progressive shrinkage, so that by adolescence
or early adulthood only a small nubbin of Fibrous
splenic tissue is left; this process is called
autosplenectomy
Q: what are the functions of spleen?
What are the surgical relevance of sickle cell disease? 1- Extramudllary hemopoiesis.
• Gall stones 2- removal of abnormal RBCs
• Autosplenectomy 3-defense mechanism against incapsulated organisms
• Avascular bone necrosis 4- synthesis of IG
• Osteomyelitis Bowel ischaemia
Cholecystitis
• Pulmonary hypertension
Avascular necrosis
• Heart failure
• Poor wound healing
Why the patient is immunocompromised?
Because of autosplenectomy with increased susceptibility of infection with encapsulated organisms

Most common organisms causing infection after splenectomy


• Streptococcus pneumoniae
• Haemophilus influenzae
• Neisseria meningitidis

Why SC patients have bone pain?


- Vaso-occlusive: triggered by infection, dehydration, acidosis, affecting the bones

65
PATHOLOGY MO’s MRCS B NOTES (Previously called Reda’s Notes) 65
What do you think of the space occupying lesion? Brain tumor
Most common brain tumor in elderly?
High-grade: Most common brain tumor of parenchymal origin?
• Gliomas and glioblastoma multiforme.
• Medulloblastoma.
Low-grade: What other brain tumors do you know?
• Meningiomas.
• Acoustic neuromas.
• Neurofibroma.
• Pituitary tumors.
• Pineal tumors.
• Craniopharyngiomas.
Secondaries

Manifestations of brain tumors?


The presentation will depend on location and rate of growth but includes features of a space occupying lesion and raised
intracranial pressure (ICP):
• Headache, which is typically worse in the mornings.
• Nausea and vomiting.
• Seizures.
• Progressive focal neurological deficits, for e.g. diplopia associated with a cranial nerve defect, visual field
defects, neurological deficits affecting the upper and/or lower limb.
• Cognitive or behavioral symptoms.
• Symptoms relating to location of mass, for e.g. frontal lobe lesions associated with personality changes,
disinhibition and parietal lobe lesions might be associated with dysarthria.
• Papilledema (absence of papilledema does not exclude a brain tumor).

What if the tumor is left untreated?


Death (Increase ICP –> herniation of brainstem –> compression of respiratory center –> Death)

Biopsy showed squamous cells?


Metastatic SCC

Possible primary sources?


SCC of skin, lung, esophagus, nasopharynx, cervix

Post biopsy had wound infection, common organisms?


Staph aureus

Wound discharge showed glucose, cause?


CSF communication

66
66 MO’s MRCS B NOTES (Previously Reda’s called Notes) PATHOLOGY
STATION 2:
Polycystic Kidney Disease (APKD)

SCENARIO:
Patient with ADPK going for bilateral nephrectomy due to intractable abdominal pain.

You are shown this picture

Describe gross pathology?


Enlargement of the kidney with multiple cyst formations

Mode of inheritance?
Autosomal dominant condition due to mutations in 2 genes: PKD1,
PKD2

Pathogenesis of cyst formation? Mutation in polycystin 1 and 2 or fibrocystin,


The cells of the renal tubules divide repeatedly until causing an out-
pocketing of the tubular wall with the formation of a saccular cyst
that fills with fluid derived from glomerular filtrate that enters from
the afferent tubule segment. Progressive expansion eventually
causes most of the emerging cysts to separate from the parent
tubule, leaving an isolated sac that fills with fluid by transepithelial
secretion. This isolated cyst expands relentlessly as a result of
continued proliferation of the mural epithelium together with the
transepithelial secretion of sodium chloride and water into the
lumen

Possible mechanisms of cyst formation in cystic kidney diseases

Other organs in the abdomen causing cyst formation?


• Liver
• Ovaries
• Pancreas
• Spleen
• Lung
• Intestines

67
PATHOLOGY MO’s MRCS B NOTES (Previously called Reda’s Notes) 67
Complications? That is not seen in the picture?
• Renal failure
• Infection
• Hypertension

Other pathology associated with PCKD?


• Liver cysts
• Pancreatic cyst
• Brain aneurysms
• Mitral valve prolapse

Associated lesion in the brain?


Cerebral aneurysm

Why there is pain?


Weight of the organ dragging upon its pedicle or stretching of renal capsule by cysts. and Infection, or hemorrhage.

What other symptoms?


• Pain
• Irregular abdominal mass
• Hematuria
• Infection
• Hypertension
• Uremia

Other differentials?
• Simple cyst
• Acquired cystic kidney disease
• VHL
• Medullary sponge kidney
• Tuberous sclerosis

Type of matching before transplant?


ABO blood matching
HLA matching: Human Leucocyte Antigens. HLA-A, HLA-B and HLA-DR are the most important. They are proteins located
on the surface of WBC.
Q: 7 days after transplant patient develop swelling in the site of operation and decline in renal function, what do you think?
- Acute rejection or infection
Types of graft rejections?
Type of rejection Key features
Hyperacute Occurs within minutes of clamp release He asked when dose it happen?
Due to pre formed antibodies*
Immediate loss of graft occurs The kidney will be swollen
Accelerated acute Occurs in first few days following surgery
Involved both cellular and antibody mediated injury
Pre-sensitisation of the donor is a common cause
Acute Traditionally the most common type of rejection
Seen days to4weeks after surgery
Predominantly a cell mediated process mediated by lymphocytes
Organ biopsy demonstrates cellular infiltrates and graft cell apoptosis
Chronic Increasingly common problem
Typically; graft atrophy and atherosclerosis are seen. Fibrosis often occurs as a late event
* Episodes of hyperacute rejection are typically due to preformed antibodies. ABO mismatch is the best example.
However, IgG anti HLA Class I antibodies are another potential cause. These events are now seen less commonly because
the cross matching process generally takes this possibility into account
Q: Can non perfectly matched kidney be transplanted? Yes
Q: How to prevent rejection? Use immunosuppression
What types of malignancy occur with immunosuppression?
Malignancy: this is 5 times greater than the normal population. Most commonly squamous cell carcinoma of skin, cervix,
basal cell carcinoma’s, lymphoma and Kaposi’s sarcoma.

68
68 MO’s MRCS B NOTES (Previously Reda’s called Notes) PATHOLOGY
STATION 10:
32 year old male, office worker, presented to outpatient department with groin
lump to the right side, this was firm and not tinder and it has been present for
one month, during your examination you notice that the patient has only one
Testicular Teratoma testes palpable at the scrotum, US report showing 6 cm partially cystic and
solid mass on the internal inguinal ring extended to the abdomen and local soft
tissues, no other intra-abdominal pathology is seen.

On ultrasound, groin mass turned out to be undescended testis with solid SCENARIO:
and cystic components (i.e. suspicious of cancer) 35y man with left groin mass,
Tissue diagnosis and you are given the following a pathology report:
Q2. What are 3 key pathology findings? examination revealed a single palpable
Teratoma testis
Positive margins (Incomplete resection)
Lymphovascular invasion (Suggesting dissemination)
T4, Nx (i.e. nodal disease can’t be assessed), Mx (metastatic disease can’t be assessed)
Q10. If this is a 60 yr/M , most common cause of non germ cell testicular neoplasm ? Lymphoma (NHL)
Define cryptorchidism?
Cryptorchidism is a complete or partial failure of the intra-abdominal testes to descend into the scrotal sac and is
associated with testicular dysfunction and an increased risk of testicular cancer.

Etiology / Risk factors of cryptorchidism


• Family history
• Low birth weight, premature birth
• High abdominal pressure (gastroschisis) Tobacco consumption
• Down syndrome
m/c location of UDT? Inguinal canal (70%) under External Oblique.
How does undescended testis contribute to increased risk of testicular cancer?
The cryptorchid testis carries a 3 to 5-fold higher risk for testicular cancer, which arises from foci of intratubular germ cell
neoplasia within the atrophic tubules
Abnormalities in the descent likely indicate an abnormality in the testicle making cancer is more likely.
Other complications?
Infertility, inguinal hernia, testicular torsion
Q . CT-TAP shows RPLN compressing on IVC. How does this
What will be your management? contribute to thrombosis? Examiner wants to hear Virchow;s triad
• CT abdomen and pelvis --- stasis,hyper coagulability, endothelial injury
• Staging
• Discuss in MDT
• Orchiectomy + chemotherapy
• If nodal disease → RPLND (Retroperitoneal Lymph Node Dissection)

Role of orchiopexy?
Reduces risk of infertility and cancer, testicle can be checked at new location (for ca)

Types of germ cell tumors? According to age:


• Seminomatous tumors
o Seminoma 30 to 40
o Spermatocytic seminoma
• Nonseminomatous tumors
o Embryonal carcinoma
o Yolk sac (endodermal sinus) tumor
o Choriocarcinoma
• Teratoma (20 to 40 years )
• Sex Cord-Stromal tumors
o Leydig cell tumor
o Sertoli cell tumor

Discuss pathology report with family in 3 simple lines?


• This is cancer in the testis
• With incomplete resection
• With lymphatic spread

Where teratoma spread to first?


To the para-aortic lymph nodes
Q9. Which other nongerm tumor my have raised AFP?
- HCC
Where does it spread to next?
Locoregional (Lungs)

69
PATHOLOGY MO’s MRCS B NOTES (Previously called Reda’s Notes) 69
Q7. Alpha fetoprotein (AFP) is usually tested as part of investigation of teratoma, what is AFP?
- protein normally produced by yalk sac in fetus, it may indicate yalk sac differentiated teratoma.
Q8. If untreated where would treatoma likely initially to spread?
- para-aortic LN >> Lungs
Serological markers / Blood tests?
Biologic markers include
• HCG
• AFP
• Lactate dehydrogenase
Which HCG? Q . Tell me about hCG : human chorionic gonadotropin
B-HCG Q . Can hCG measure seminoma recurrence ? . Yes.
Other conditions where HCG is elevated?
Pregnancy

Not asked Value of serum markers is fourfold


• In the evaluation of testicular masses
• In the staging of testicular germ cell tumors. For example, after orchiectomy, persistent elevation of HCG or AFP
concentrations indicates stage II disease even if the lymph nodes appear of normal size by imaging studies.
• In assessing tumor burden
• In monitoring the response to therapy. After eradication of tumors there is a rapid fall in serum AFP and HCG.
With serial measurements it is often possible to predict recurrence before the patients become symptomatic or
develop any other clinical signs of relapse.

Q: Post-operative developed hematoma, mention stages of hematoma resolution?


• Lysis of the clot by macrophages (about 1 week)
• Growth of fibroblasts from into the hematoma (2 weeks)
. • Hyalinzed tissue formation.
After few months developed small pneumothorax, what’s the cause?
Lung metastasis

Define metastasis?
Survival and growth of cells at a site distant from their primary origin

Q14. One year later the patient came with para-aortic lymph node compressing renal artery and vein + SOB + PE
Why PE in this patient? Q12. PE composed of what? Platelets + polymerized fibrin + admixed cells
• Venous stasis (due to mass effect around renal vessels)
• Hypercoagulable state (Alterations of clotting factors because of tumor itslf)
Why hypercoagulable state?
Tumor cells to produce and secrete procoagulant/fibrinolytic substance which activate coagulation cascade
stimulation of tissue factor production by host cell.
Q13. What clotting factor that allow polymerization of fibrin ? Activated factor 8
Q15.Which part of Virchow's triad is missing here?
Endothelial injury (Alterations of blood constitute)

Q3. What is choriocarcinoma and what is its tumor marker? Malignant teratomatus neoplasm with placenta-like differentiation.
Choriocarcinoma is a highly malignant form of testicular tumor.
Histologically the tumors contain two cell types, syncytiotrophoblast and cytotrophoblast. Tumor marker is B-HCG
Q4. What you will find in IHC to tell that is choriocarcinoma?
- B-HCG
Q5. What is the common tumor in this age group?
Seminoma
(Non–Hodgkin lymphoma is the most common testicular tumor in men older than 60 years.)
Q6. If the pt is 20 what will affect the diagnosis?
- Teratoma
What is the cell origin of seminoma?
Most testicular germ cell tumors originate from a precursor lesion called intratubular germ cell neoplasia (ITGCN)

Q11. Histopath showed papillary thyroid tissue and GIT adenocarcinoma, why?
Teratoma has the 3 germ cell lines (mesoderm, endoderm, ectoderm)
Q. Gave pathology report : teratoma. What is teratoma ?
a tumor with tissue or organ components resembling normal derivatives of more than one germ layer.

Q. What is the overall prognosis of testicular cancer?


Prognosis depends on tumour stage. 5-year survival ranges from 92-94% for patients with good prognostic features to 50% for
patients with poor prognostic features.

70
70 MO’s MRCS B NOTES (Previously Reda’s called Notes) PATHOLOGY
STATION 1:

First stem:
Second stem:
Prostate Cancer
65 year old man referred by his GPsith increasing poor urinary stream with nocturea, he had stroke last year, he has atrial fibrillation need warfarin treatment.

SCENARIO: A man with BPH, poor urine stream, low back pain

DRE was done and a firm, round prostate was felt, what’s your diagnosis
BPH
DRE is undertaken in the clinic to assess the size of the prostate, what is the reason for prostate growth during life?
Cause of prostate enlargement?
Androgen hormone stimulation of glands and stromal growth.
Q: Features in DRE sugges PC? Hard, Fixed to rectal wall, Nodular, obliterated median sulcus
Investigations (Name 3 pathological investigations that can be performed in the outpatient clinic to aid the diagnosis)
PSA
Mid-Stream Urine analysis
U & Es and creatinine

!

How to take a prostate biopsy?
TRUS guided from: mid lobe parasagittal plane at: the apex, the mid gland, and the base, bilaterally.

Why multiple biopsies?


Answer only: • Prostatic cancer can be focal, so it’s important to take samples from different sites
• There is often only a scant amount of tissue available for histologic examination in needle biopsies
• Malignant glands may be admixed with numerous benign glands
• Moreover, the histologic findings pointing to malignancy may be subtle (leading to underdiagnoses),
• There are also benign mimickers of cancer that can lead to a misdiagnosis of cancer
• Sensitivity of needle biopsy could improve by 30 % to 35 % by increasing the number of biopsy cores beyond 6 (14 to 45).
How to differentiate between rectum and prostate cells in a needle biopsy?
• Using immunohistochemical marker (α-methylacyl-coenzyme A-racemase) (AMACR)
• CEA (in rectal cells) & PSA

Not asked What are the gene mutations involved in pathogenesis of prostate cancer? (BRCA2, BRCA1, ATM, HOXB13, CHEK2, MSH2, MLH1)
T
h
e
m
o
s
t
c
o
m
m
o
n
a
c
q
u
i
r
e
d
g
e
n
e
t
i
c
l
e
s
i
o
n
s
i
n
p
r
o
s
t
a
t
i
c
c
a
r
c
i
n
o
m
a
s
a
r
e
T
M
P
R
S
S
2
-
E
R
G
f
u
s
i
o
n
g
e
n
e
s
a
n
d
“Patient went to radical prostatectomy, PSA is 7 three months later”
m
u
t
a
t
i
o
n
s
o
r
d
e
l
e
t
i
o
n
s
t
h
a
t
a
c
t
i
v
a
t
e
t
h
e
P
I
3
K
/
A
K
T
s
i
g
n
a
l
i
n
g
p
a
t
h
w
a
y
.
How can you judge the success of radical prostatectomy?
Fall in the level of PSA below detectable levels within 4-6 weeks
High PSA after prostatectomy → recurrence should be considered

Why PSA is not reliable?


• PSA is organ specific, but not cancer specific.
• Although serum levels of PSA are elevated to a lesser extent in BPH than in prostatic carcinomas, there is
considerable overlap.
• Other factors such as prostatitis, infarction of nodular hyperplasia, instrumentation of the prostate, and
ejaculation also increase serum PSA levels

His symptoms responded to treatment but a repeat PSA 6 months later is 12 ng, his mid-stream urine is negative what is the
significance of PSA results?
- Raised possibility of prostatic neoplasm

The biopsy describe several areas of tissues, some showing high grade prostatic neoplasia and there is some admixed rectal glandular tissue seen,
what tests can be applied to sample to confirm this non-prostatic elements?
- IHC for colon antigens should be positive
- Tissue should be PSA negative.

71
PATHOLOGY MO’s MRCS B NOTES (Previously called Reda’s Notes) 71
Biopsy showed focal well differentiated adenocarcinoma, what grading scheme is commonly used?
What is the grading system?
Gleason score
It grades prostate tumors from 2 – 10,
10 being the most abnormal and
therefore the most likely to spread.
The pathologist allocates a number
from 1 - 5 for the most common
histological pattern in the specimen,
then does the same for the second
most common pattern.
The sum of these two numbers gives
the Gleason score
(Which Biochemistry marker)
One test to exclude bony metastasis?
Alkaline phosphatase

What kind of metastases?


Sclerotic
Why?
Due to increased bone
deposition due to increased
osteoblastic activity
CT scan revealed increase density of lumbar vertebrae, why the bone is dense? The tumor induces new bone formation due to osteoblastic activity (Sclerotic).
Post-operative, developed fever, dusky red urine, why?
Post-operative, patient is confused and has a raised white cell count, why?
UTI

Which of the blood components will rise? Types of bone metastases


WBC's - neutrophils Bone metastases are typically characterized as ‘lytic’,
Most common organisms? ‘sclerotic’ or ‘mixed’, according to the radiographic
E-coli and/or pathologic appearance of the lesions.
When thinking of UTI what is the level of colony forming units is generally taken to indicate likely sepsis?
- if more than 100 thousand, if less means contamination.
One year after surgery, PSA is 3 ng/ml, Why dose this level of PSA indicate tumor recurrence from the surgery described?
PSA is only secreted by prostatic tissues. If all prostatic tissue was removed, PSA should not be present

What is the rational in treating prostate cancer by bilateral orchidectomy?


Androgen deprivation, So reducing the rate of cancer growth.
The growth and survival of prostate cancer cells depends on androgens, which bind to the androgen receptor (AR) and
induce the expression of pro-growth and pro-survival genes.
Which testicular parenchymal cells are responsible from androgen synthesis and secretion?
What are the cells producing testosterone?
Leydig cells

Lobes of the prostate

72
72 MO’s MRCS B NOTES (Previously Reda’s called Notes) PATHOLOGY
STATION 9:
Abscess

Definition? SCENARIO:
Focal collection of pus that may be caused by seeding of pyogenic organisms into a Patient with forearm abscess
tissue or by secondary infections of necrotic foci.

What tissue is the wall of an abscess characteristically


composed of?
Granulation tissue. This used to be called the “pyogenic
membrane”, but it is not a true membrane and is not
itself pyogenic.

What is present inside an abscess?


Central, largely necrotic region rimmed by a layer of
preserved neutrophils, with a surrounding zone of
dilated vessels and fibroblast proliferation indicative of
attempted repair.

What is pus?
Pus is the product of acute inflammation composed of
cellular and fluid, exudative phases.

Organisms causing abscess? The abscess contains neutrophils and cellular debris and is
• Bacterial: Staph aureus, Streptococcus surrounded by congested blood vessels.
pyogenes
• Non-bacterial: Fungal, viral, parasitic

One simple test to detect the cause of abscess?


Gram stain

What is the cause of fever in abscess?


Fever is produced in response to substances called pyrogens (TNF, IL-1) that act by stimulating prostaglandin synthesis in
the vascular and perivascular cells of the hypothalamus.

Major roles of cytokines in acute inflammation.

73
PATHOLOGY MO’s MRCS B NOTES (Previously called Reda’s Notes) 73
Define cellulitis?
A spreading bacterial infection of the skin affects the dermis and subcutaneous fat, characterized by redness, warmth,
swelling, and pain

What is the difference between abscess and cellulitis?


Abscess Cellulitis
Caused by Staphylococcus aureus Group A Streptococci
Causative organism Organism secretes coagulase enzyme Fibrinolysin
Hyaluronidase
Inflammation Localized Diffuse
Necrosis Central zone of necrosis Extensive necrosis
Pus Rapid formation, thick and yellow Slow formation, thin and contains RBCs

When to give Antibiotic in abscess?


Antibiotics are indicated if the abscess is not localised (e.g. evidence of cellulitis) or the cavity is not left open to drain
freely

What are giant cells?


These are multinucleated cells derived from the fusion of multiple activated macrophages to form granuloma

Most common cause of granuloma formation?


Tuberculosis

Causative agent?
Mycobacterium T.B

Type of stain?
Ziehl–Neelsen stain (Acid fast stain)

What changes you see in lymph node affected with


inflammation?
Reactive follicular hyperplasia

Typical tuberculous granuloma showing an area of central necrosis


surrounded by multiple Langhans-type giant cells, epithelioid cells,
and lymphocytes.

74
74 MO’s MRCS B NOTES (Previously Reda’s called Notes) PATHOLOGY
STATION 5:
Osteomyelitis

SCENARIO: Patient with previous tibial fracture fixation 3 years ago and presenting with redness and swelling of knee

Common organisms causing osteomyelitis?


Age group Most common organisms
Newborns (< 4 m) S. aureus, Enterobacter species, and group A and B Streptococcus species
Children (4 m to 4 y) S. aureus, group A Streptococcus species, Kingella kingae, and Enterobacter species
Children, adolescents S. aureus (80%), group A Streptococcus species, H. influenzae, and Enterobacter species
(4 y to adult)
Adult S. aureus and occasionally Enterobacter or Streptococcus species
Sickle Cell Anemia S. aureus is typically most common, but Salmonella species is pathognomonic
Patients

About Osteomyelitis

Osteomyelitis can be classified on the basis of patient age (pediatric or adult), causative organism, pathogenesis
(contiguous spread, traumatic, hematogenous), anatomic location, or duration of symptoms (acute, subacute,
chronic). These variables can be used individually or in combination for categorization. There are also a number of
named classification systems that focus on various clinical aspects of osteomyelitis, but no one system is universally
accepted. The most commonly used classification system for adult osteomyelitis is the Cierny–Mader staging system

Cierny Classification of Osteomyelitis


(describes anatomic involvement, host, treatment, prognosis)
Anatomic Location
Stage I Medullary
Stage 2 Superficial
Stage 3 Localized
Stage 4 Diffuse
Host Type
Type A Normal
Type B Compromised
Type C Treatment is worse to patient than infection

Not
important
Exogenous: most common osteomyelitis in adults
• Acute osteomyelitis from open fracture or bone exposed at surgery
• Chronic osteomyelitis from neglected wounds: diabetic feet, decubitus ulcers
Hematogenous: most common osteomyelitis in children
• Bloodborne organisms of sepsis (often positive blood cultures before antibiotics given)
• Pediatric—immature immune system
o Metaphysis or epiphysis of long bones
o Lower extremity more often than upper
o Boys more often than girls
• Adults—immunocompromised—vertebrae most common adult hematogenous site
o Dialysis patient—rib and spine osteomyelitis
o IV drug abuser—medial or lateral clavicle osteomyelitis
o Elderly, IV drug abuser, transplant patients

Subacute osteomyelitis: Brodie abscess


Residual of acute osteomyelitis versus hematogenous seeding of growth plate trauma

75
PATHOLOGY MO’s MRCS B NOTES (Previously called Reda’s Notes) 75
Pathogenesis of osteomyelitis?
1- Invasions and Inflammation
2- Suppuration
3- Necrosis (sequestration)
4- New bone formation
5- Resolution

76
76 MO’s MRCS B NOTES (Previously Reda’s called Notes) PATHOLOGY
What is abscess? See previous stations (Carcinoid station and Abscess station)
Is a localized painful collection of pus in tissues, organs, or confined spaces usually because of an infection by a pyogenic
organism. It is surrounded by granulation tissue called pyogenic membrane, however it is not a true membrane and is
not itself pyogenic.

What is pus?
Thick, yellowish liquid that is formed as part of an inflammatory response typically associated with an infection and is
composed of exudate chiefly containing dead white blood cells (as neutrophils), tissue debris, and pathogenic
microorganisms (as bacteria)

Pathogenesis of pus formation?


Microbial invasions → acute inflammation with vascular congestion and exudation of fluids with infiltration of PMN cells

Fate of abscess
• Resolution
• Rupture
• Spread - sepsis
• Chronic abscess formation

Definition of sequestrum?
Dead bone that has become separated during the process of necrosis from normal or sound bone. It is a complication
(sequela) of osteomyelitis

Definition of involucrum?
Reactive woven or lamellar bone depositions forming a shell of living tissue around a sequestrum
Or Thick sheath of periosteal new bone surrounding a sequestrum.

Why pus may burst through the bone?


• Due to increased intraosseous pressure due to increased osmolarity which occurs due tissue breakdown
• Due to formation of a large rounded cloacal opening in the involucrum. So, pus will escape through the
involucrum and the surrounding soft tissue to the skin surface within a sinus tract

Why the fixing plate should be removed? What is the pathological process of puss coming out of the plate?
- Chronic osteomyelitis
• It has become a septic focus
• Formation of biofilm - less sensitive to antibiotics
• Implant might be loose

SCC developed in the sinus, why?


Due to chronic irritation

In chronic osteomyelitis what is the sequence of events by which the draining sinus can develop SCC
Chronic irritation → hyperplasia → dysplasia → carcinoma
Metaplasia

Name of protein deposition in chronic OM? (In acute >> CRP)


Amyloid AA

Treatment?
• Antibiotic therapy: After taking sample for C&S and according to hospital policy after discussion with microbiologist
o Blood cultures are taken and high-dose intravenous antibiotics, active against Staphylococcus
aureus, Streptococci and Gram-negative rods such as Escherichia coli are given.
o Cephalosporins, co-amoxiclav or a combination of Flucloxacillin and Gentamicin may be used.
• Supportive treatment for pain and dehydration
• Splintage of the limb
• Surgical drainage: if there is no response to antibiotics for 2 days

77
PATHOLOGY MO’s MRCS B NOTES (Previously called Reda’s Notes) 77
Differential diagnosis of swollen knee?
• OA and overuse syndrome
• Septic arthritis, infections (gonorrhea, Lyme disease, TB, brucellosis)
• Gout
• Pseudogout (Ca+2 pyrophosphate deposition disease)
• Hemarthrosis
• Tumors
• Trauma (ligamentous injury, fractures, patellar dislocation, meniscus injury, etc.)
• Polyarthritis (RA, Reiter’s syndrome, Juvenile rheumatoid arthritis)

Single bedside test to do?


Aspiration

Tests to be done on the aspirate?


• Urgent gram stain
• Cytology, chemical analysis, crystals
• CS (culture & sensitivity)

Types of crystals in gouty arthritides?


Characteristic Gout Pseudogout
Crystal composition Monosodium urate Calcium pyrophosphate
Crystal shape Needle-shaped Rhomboid-shaped
Birefringent Negative Weakly Positive
Most common site 1st MTPJ Knee
Radiography “Rat-bite” erosions White lines of chondrocalcinosis
1st line treatment NSAIDs

78
78 MO’s MRCS B NOTES (Previously Reda’s called Notes) PATHOLOGY
STATION 4:
Pathological Fractures

What is pathological fracture? SCENARIO:


Bone fracture which occurs without adequate trauma and is caused by a Young female was cycling
preexistent pathological bone lesion. developed pain in her thigh
soon as she put her foot on the
Causes? ground. X-ray showed fracture
• Neoplastic: malignant tumors shaft femur with hypodense
o Primary (multiple myeloma) shadow
o Metastatic
• Non-neoplastic:
o Osteoporosis
o Osteomyelitis
o Osteomalacia
o Osteogenesis imperfecta
o Paget’s disease
o Bone cyst
o Radiotherapy

Mention 5 possible primary sites for metastasis?


• Breast 2s above you chest (Breast, lungs, thyroid)
Urine track (kidney and prostate)
• Prostate (Most common)
• Lung
• Thyroid Q: Brain tumor casing sclerotic metastasis to bone?
• Kidney - Medulloblastoma

Fixation was done, how to check malignancy? (During internal fixation)


Bone biopsy

Pathology report: typical bland appearance (follicular cells), what is the probable location primary tumor?
Thyroid gland

We have done FNAC of the thyroid but unable to differentiate cancer, why? In follicular thyroid neoplasm,
Malignancy is determined by capsular and vascular invasion which need histology rather than a cytology to confirm

What investigations to do to confirm thyroid metastasis?


Radioactive iodine scan
W
h
a
t
a
r
e
o
t
h
e
r
s
i
t
e
s
o
f
e
c
t
o
p
i
c
t
h
y
r
o
i
d
t
i
s
s
u
e
s
o
t
h
e
r
t
h
a
n
h
e
a
d
a
n
d
n
e
c
k
a
n
d
t
h
o
r
a
x
?
O
v
a
r
i
a
n
a
n
d
t
e
s
t
i
c
u
l
a
r
t
e
r
a
t
o
m
a
79
PATHOLOGY MO’s MRCS B NOTES (Previously called Reda’s Notes) 79
Which thyroid cancer will show no response to iodine uptake?
Medullary thyroid cancer as its origin is from parafollicular C cells so it is not of a follicular origin

If a patient is O +ve, what test to do prior to blood transfusion?


• ABO
• RH group cross matching

If the patient is telling you his group, you will still do cross matching and why?
Yes, to determine if the recipient has pre-formed antibodies against any antigens on the donor's cells

Complications of incompatible blood transfusion?


• Acute hemolytic reactions
• Febrile non- hemolytic transfusion reaction
• Delayed hemolytic transfusion reactions
• Transfusion related acute lung injury

Define hemolysis
Rupture (lysis) of RBCs and release of their contents (cytoplasm) in the surrounding fluid (blood, plasma)

STATION 10:

80
80 MO’s MRCS B NOTES (Previously Reda’s called Notes) PATHOLOGY
Polytrauma + Transfusion
Patient have K wire and wound start oozing
Q: Define DIC? SCENARIO:
A patient who is known to
have hepatitis C → trauma →
blood loss
Splenectomy was done and
• Widespread hemorrhage patient received blood
I
t
i
s
a
p
a
t
h
o
l
o
g
i
c
a
l
c
o
n
s
u
m
p
t
i
v
e
c
o
a
g
u
l
o
p
a
t
h
y
d
u
e
t
o
a
c
t
i
v
a
t
i
o
n
o
f
t
h
e
c
o
a
g
u
l
a
t
i
o
n
• Thrombocytopenia, decreased fibrinogen, increased FDPs transfusion
a
n
d
f
i
b
r
i
n
o
l
y
t
i
c
s
y
s
t
e
m
s
,
w
h
i
c
h
l
e
a
d
s
t
o
f
o
r
m
a
t
i
o
n
o
f
m
i
c
r
o
t
h
r
o
m
b
i
i
n
m
a
n
y
o
r
g
a
n
s
(Fibrin degradation product) Patient develops DIC
w
i
t
h
t
h
e
c
o
n
s
u
m
p
t
i
o
n
o
f
t
h
e
c
l
o
t
t
i
n
g
f
a
c
t
o
r
s
a
n
d
p
l
a
t
e
l
e
t
s
.
Functions of the platelets?
C
h
a
r
a
c
t
e
r
i
z
e
d
b
y
:
Platelets contribute to the hemostatic process in two different ways. First, through their adhesive and cohesive functions
that lead to the formation of a hemostatic plug. Second, they can activate coagulation mechanisms

What is the precursor of platelets? How platelets are formed from bone marrow?
From megakaryocytes by fragmentation.
Extra questions:
Shelf life? 5 days
Stages of hemostasis #How to correct coagulopathy in this patient (DIC’?
See ASSCC, Ruptured AAA/Hypothermia station - Blood, platelets, FFP, cryoprecipitate.

# what else can assist in correcting this patient bleeding?


Why this patient has bleeding tendency? - warming the patient by fluid warmer or forced hot air.
Because of the liver affection
# as part of prevention of hypothermia, what other intraoperative needs to decrease
transfusion of donated blood?
What are very late manifestations of HCV? - Auto-transfusion
• Cirrhosis
• HCC

What activates intrinsic and extrinsic pathways? and exposed endothelial collagen
• Intrinsic pathway is activated by vessel injury which will lead to activation of factor 12 >> aPTT
• Extrinsic pathway is activated by tissue thromboplastin released by the damaged cells >> PT
• APTT tests for intrinsic pathway (tissue factor)
• PT tests for extrinsic pathway and the common pathway

Hypersensitivity reactions: is exaggerated or inappropriate immunologic responses occurring in response to an antigen or allergen.
The Gell and Coombs classification divides hypersensitivity reactions into 5 types
Type I Type II Type III Type IV Type V
Description Anaphylactic Cytotoxic Immune complex Delayed type Autoimmune
Mediator IgE IgG, IgM IgG, Ig A, IgM T-cells
Antigen Exogenous Cell surface Soluble Tissues
Response time Minutes Hours Hours 2-3 days
Examples Asthma Autoimmune hemolytic anaemia Serum sickness Graft versus host disease Mythena
gravies
Hay fever Pemphigus SLE Contact dermatitis
Goodpasture's Aspergillosis

Which blood product will you give? Mnemonic for the reactions and the mediators involved ACID EGG-T
Packed RBC's ACID EGG T (mediators)
• Type 1 Anaphylactic • IgE
Percentage of white blood cells in packed • Type 2 Cytotoxic • IgG
RBC's? • Type 3 Immune complex • IgG
<5x10^6 cells/L (Leukoreduced Packed RBC's) • Type 4 Delayed type • T cells
- LPRC
Q: Platelet in packed RBCs? 0%
What’s the life span of RBC's?
Q: How to reduce risks of blood transfusion reaction or transfusion of whole blood?
120 days #preOp:
- Consider the level of which blood products are required.
What tests to do before blood transfusion: - Use hematenics and erythropoietin to correct anemia before operation.
#postOp:
• ABO - Avoid blood loss by use of diathermy and good hemostasis.
• RH - Use plasma expanders like crystalloids and colloids.
- Autologous transfusion.
What to tell hematologist before giving blood?
- Grouping and coss matching (CXM)

81
PATHOLOGY MO’s MRCS B NOTES (Previously called Reda’s Notes) 81
What is GXM: A group and save is the sample processing. It
Group cross matching: to test donor red cells against recipient serum to consists of a blood group and an antibody
detect any potential incompatibility through which antibodies in recipient screen to determine the patient’s group and
cause hemolysis to donor cells whether or not they have atypical red cell
antibodies in their blood. If atypical
Antigen in cross matching? antibodies are present the laboratory will do
additional work to identify them.
ABO and RH
A crossmatch is when the laboratory actually
What are the stages of bone healing? with duration? provides red cells products for the patient. It
1. Hematoma formation (mass of clotted blood) at fracture site. is not possible for the laboratory to provide
Tissue in fracture site swells, very painful, obvious inflammation, crossmatched blood without having
and bone cells are dying. In first day processed a group and save sample first.
2 to 3 weeks
2. Fibrocartilaginous callus develops over a 3 to 4-week period.
This process involves
• Capillary growth in the hematoma
• Phagocytic cells invading and cleaning-up debris in injury site
• Fibroblasts and osteoblasts migrating into site and beginning reconstruction of bone
Note that the fibrocartilaginous callus serves to splint the fracture.
3. Bony callus begins forming after 3 to 4 weeks after injury and is prominent 2 to 3 months following the injury.
Continued migration and multiplying of osteoblasts and osteocytes result in the fibrocartilaginous callus turning
into a bony callus.
4. Remodeling. Any excess material of the bony callus is removed and compact bone is laid down in order to
reconstruct the shaft. Remodeling is the final stage. 3 months to 1 year

Effects of prolonged immobility on bone?


Loss of bone density and increased risk of osteoporosis (Results in hypercalcemia)

Infected implant, why you should remove?


Septic focus + become loose + form a biofilm

What the 1st test to do?


Wound swab for culture and sensitivity

What is PVL Staph aureus? Pore forming


Panton-Valentine leukocidin (PVL) is a cytotoxin produced by Staphylococcus aureus that causes leukocyte destruction
and tissue necrosis.
It is one of the β-pore-forming toxins. The presence of PVL is associated with increased virulence of certain strains
(isolates) of Staphylococcus aureus. It is present in the majority of community-associated Methicillin Resistant
Staphylococcus aureus (CA-MRSA)

What is the effect of this cytotoxin?


PVL creates pores in the membranes of infected cells and is the cause of necrotic lesions involving the skin or
mucosa, including necrotic hemorrhagic pneumonia.(necrotizing pneumonia)

82
82 MO’s MRCS B NOTES (Previously Reda’s called Notes) PATHOLOGY
STATION 11:
Hepatitis C (HCV)
Define SCENARIO:
Hepatitis C?
Is inflammation that disrupts hepatocytes and small bile ductules that is caused by virus 55-year-old lady, IVDU with
C via parenteral transmission (e.g., IVDA, unprotected intercourse, needle stick) left lower limb venous ulcer
risk from transfusion is almost nonexistent due to screening of blood

Type of Hepatitis C virus?


Is a single-stranded RNA virus from family flaviviruses
• What is the clinical feature of HCV?
• Acute hepatitis presents as jaundice (mixed CB and UCB) with dark urine (due to CB), fever, malaise, nausea, and
elevated liver enzymes (ALT > AST)
• Chronic hepatitis is characterized by symptoms that last > 6 months. With a risk of progression to cirrhosis &
HCC
The leading causes of chronic liver failure
What is the pathological sequence in HCV worldwide include chronic hepatitis B, chronic
1. Acute Hepatitis hepatitis C, nonalcoholic fatty liver disease, and
2. Chronic Hepatitis alcoholic liver disease.
3. Liver cirrhosis & portal hypertension
4. Liver cell failure
5. HCC

Most common risk factors for HCV infection?


• Intravenous drug abuse
• Multiple sex partners
• Having had surgery within the last 6 months
• Needle stick injury
• Multiple contacts with an HCV-infected
individual
• Employment in the medical or dental field
Define
Cirrhosis
End-stage liver damage characterized by disruption of
the normal hepatic parenchyma by bands of fibrosis
and regenerative nodules of hepatocytes

Clinical features & Pathological sequence?


• Portal hypertension
o Ascites
o Congestive
splenomegaly/hypersplenism
o Portosystemic shunts (esophageal
varices, hemorrhoids, and caput
medusae)
o Hepatorenal syndrome (rapidly
developing renal failure secondary to
cirrhosis)
• Decreased detoxification
o Mental status changes, asterixis, and
eventual coma (due to raised serum
ammonia)
o Gynecomastia, spider angiomas and
palmar erythema due to hyperestrinism
o Jaundice
• Decreased protein synthesis
o Hypoalbuminemia and edema Major clinical consequences of portal hypertension in the setting of
o Coagulopathy due to decreased cirrhosis, shown for the male. In women, oligomenorrhea, amenorrhea,
synthesis of clotting factors. and sterility as a result of hypogonadism are frequent.
• Hepatocellular carcinoma

Q: Mechanism of portal hypertension?


Liver cirrhosis lead to increase resistance of blood flow through the liver

83
PATHOLOGY MO’s MRCS B NOTES (Previously called Reda’s Notes) 83
The most common cause of cirrhosis in the UK?
Chronic Alcoholism

Alcoholic liver disease. The interrelationships


among hepatic steatosis, alcoholic hepatitis,
and alcoholic cirrhosis are shown, along with
depictions of key morphologic features. It
should be noted that steatosis, alcoholic
hepatitis, and steatofibrosis may also
develop independently. In particular some
patients present initially with cirrhosis
without any of the other forms of alcoholic
liver disease.

The most common cause of death in cirrhosis?


Rupture esophageal varices

Define Necrosis:
Necrosis is a form of cell death in which cellular
membranes fall apart, and cellular enzymes leak
out and ultimately digest the cell
Q: Divided into several types based on gross features With examples
• Coagulative necrosis: ischemic infarction
of any organ except the brain
• Liquefactive necrosis: Brain infarction,
Abscess, Pancreatitis
• Gangrenous necrosis: dry gangrene, wet
gangrene
• Caseous necrosis: TB
• Fat necrosis: Traumatic in breast,
Enzymatic in Pancreatitis
• Fibrinoid necrosis: Hypertension
See Gangrene + Mesothelioma station

What is the type of necrosis in HCV?


Coagulative

Q: What is the type of necrosis in Venous ulcer? (GLCer)


Coagulative, Liquefactive and Gangrenous.

Mechanisms of fibrosis. Persistent tissue injury leads to chronic


inflammation and loss of tissue architecture. Cytokines produced by
macrophages and other leukocytes stimulate the migration and
proliferation of fibroblasts and myofibroblasts and the deposition of
collagen and other extracellular matrix proteins. The net result is
replacement of normal tissue by fibrosis.

84
84 MO’s MRCS B NOTES (Previously Reda’s called Notes) PATHOLOGY
Types of Candida? yeast, pseudohyphae, or true hyphae. Candida albicans is the most
• Oral common disease-causing
• Vaginal fungus. It is a normal
• Cutaneous inhabitant of the oral cavity,
• Invasive gastrointestinal tract, and
Oral candida albicans is the most common type (oral thrush) it can grow as yeast, vagina in many individuals.
pseudohyphae, or true hyphae Systemic candidiasis (with
associated pneumonia) is a
disease restricted to
immunocompromised patients
that has protean
manifestations.

In tissue sections, C. albicans


demonstrates yeastlike forms
(blastoconidia), pseudohyphae,
and true hyphae

Pseudohyphae
Are an important diagnostic
clue and represent budding
yeast cells joined end to end at
constrictions, thus simulating
true fungal hyphae.
The organisms may be visible
with routine H&E stains, but a
variety of special “fungal”
stains (Gomori
The morphology of fungal infections. (A) Candida organism has pseudohyphae and budding
methenamine-silver, periodic
yeasts (silver stain). (B) Invasive aspergillosis (gross appearance) of the lung in a hematopoietic
acid–Schiff) commonly are used
stem cell transplant recipient. (C) Gomori methenamine-silver (GMS) stain shows septate
hyphae with acute-angle branching, consistent with Aspergillus. (D) Cryptococcosis of the lung to better highlight the
in a patient with AIDS. The organisms are somewhat variable in size. pathogens.

Q: What type of LN changes in HCV related lymphadenopathy?


Follicular hyperplasia.

Q: Where these changes happened in LN?


In cortex.

85
PATHOLOGY MO’s MRCS B NOTES (Previously called Reda’s Notes) 85
Q: Give me 4 pathological process for venous ulcer in IVDU?
- trauma from repeated injection
- irritative quantities of abused drugs
- venous hypertension > edema >
What are the stages and classification of venous leg ulcers? - thrombosis and thrombophlebitis.
CEAP classification
Q: what are complication of blood transfusion?
• C0 – No visible or palpable signs of venous disease
• C1 – Telangiectasias or reticular veins Q: define massive blood transfusion?
• C2 – Varicose veins
Q: what are complication of massive blood transfusion?
• C3 – Edema Q: How to deferential between necrotizing faciatis and cellulitis?
• C4a – Milder skin changes due to venous disorder (pigmentation, eczema)
• C4b – Severe skin changes due to venous disorder (dermatosclerosis, atrophie blanche)
• C5 – Healed ulcers
• C6 – Skin changes with active ulcers

Q: Causes of inguinal lymphadenopathy?


Systemic
• Neoplastic
o Hodgkin’s and non-Hodgkin’s
lymphoma (common)
o Leukemia (uncommon
• Inflammatory
o Tuberculosis (uncommon)
o Sarcoid (uncommon)
Local causes
• Infective
o Non-specific reactive lymphadenopathy
form infection in groin or lower limb
(very common)
• Specific infections
o Cat scratch fever (very rare)
o Lymphogranuloma venereum (very
rare)
• Neoplastic
o Metastatic disease (from primary lesion
I lower limb, external genitalia, perianal
region)
o Malignant melanoma

86
86 MO’s MRCS B NOTES (Previously Reda’s called Notes) PATHOLOGY
Q: What you will see microscopically in inguinal LN cut section in this patient? Follicular hyperplasia
Chronic non-specific reactive lymphadenopathy (One of the three patterns depending on the is caused by stimuli that
causative agents) activate B cell follicles,
• Follicular hyperplasia In the cortex e.g. RA, HIV
Predominantly B-cells response with germinal center hyperplasia which may be
associated with marginal zone hyperplasia. Follicles vary in size and shape (vs Paracortical hyperplasia
lymphoma). caused by stimuli that
o collagen vascular disease trigger T-cell–mediated
o systemic toxoplasmosis immune responses, e.g.
o syphilis EBV
• Paracortical (interfollicular) hyperplasia
Reactive changes within the T-cell region of LN with paracortical expansion caused by Sinus histiocytosis
o viral infection e.g. infectious mononucleosis (EBV) refers to an ↑ in the no.
o certain vaccination (e.g., smallpox) and size of the cells that
o immune reaction induced by certain drugs line lymphatic sinusoids.
• Sinus histiocytosis (reticular hyperplasia) It is nonspecific but may
Distension and prominence of lymphatic sinusoids, due to marked hypertrophy of be prominent in LNs
lining endothelial cells and an infiltrate of macrophages (histocytes). It is often draining cancers
encountered in
o LN draining cancers
o Immune response to tumor or its products

Paracortical hyperplasia, identified by the prominence of


postcapillary venules

Follicular Hyperplasia.
A, Low-power view showing marked differences in size of
germinal centers, their well-circumscribed character, and the fact
that they are surrounded by a well-defined mantle.
B, High-power view showing numerous “tingible body”
macrophages.

Sinus Hyperplasia. The cells present in the sinus represent an


admixture of histiocytes and sinus lining cells.

87
PATHOLOGY MO’s MRCS B NOTES (Previously called Reda’s Notes) 87
STATION 11:
Acute Pancreatitis

See Acute Pancreatitis station, ASSCC SCENARIO:


What is amylase and what is its function? 50-year-old patient
complaining of epigastric pain.
• Group of proteins (digestive enzymes) found in saliva and pancreatic juice
Recent history of
• All are glycoside hydrolases and act on α 1,4-glygoside bond to convert starch
hospitalization for acute
into smaller carbohydrate molecules (oligosaccharides and disaccharides)
pancreatitis 3 weeks ago.
What is CRP and from where it produced?
• C-reactive protein (CRP) is a substance produced by the liver early in response to inflammation (acute phase
reactant)
• It is not very specific, it isn’t unique to one disease
• Can help to monitor disease progress and flares.

What is blood test to do in acute pancreatitis?


• Serum amylase
• Serum lipase (preferred over the amylase due to higher sensitivity)
• LDH
• CRP
• FBC

What biochemical test to do?


• Electrolytes
• ABG
• Ca+2
• Albumin

The patient having a compression over stomach, what is that?


Pseudocyst

What non-bacterial organism can cause infection in this patient?


• Viruses (e.g. Mumps, Coxsackie B and hepatitis)
• Parasites (e.g. ascaris lumbricoides, fasciola hepatica, and hydatid disease)
• Bacteria (mycoplasma pneumonia and leptospirosis)

Patient’s haemoglobin is dropped, why?


• Rupture of splenic artery (GDA, PDA) pseudoaneurysm
• Haemorrhage

What non-surgical option to stop bleeding?


Interventional radiology By Coiling

How does it work?


Using real-time imaging, the physician guides a catheter
through the artery and then releases clotting agents (coils,
particles, gel, or foam) into the blood vessel slowing the
blood flow and causing an internal injury to activate the
coagulation cascade.

88
88 MO’s MRCS B NOTES (Previously Reda’s called Notes) PATHOLOGY
Which part of coagulation will be activated?
Intrinsic pathway

What factors involved in intrinsic pathway?


(Factors XII, XI, IX, X)

89
PATHOLOGY MO’s MRCS B NOTES (Previously called Reda’s Notes) 89
STATION 15:
Summary of Definitions

Atherosclerosis: Frozen section:


Pathological process of the vasculature in which an Is a pathological labora1ory procedure to perform rapid
artery wall thickens as a result of accumulation of fatty microscopic analysis of a specimen.
materials such as cholesterol.
Gangrene:
ARDS: A type of necrosis caused by a critically Insufficient
Diffuse alveolar damage and lung capillary endothelial blood supply.
Injury.
Giant cells:
Barret's esophagus: Multinucleated cells derived from the fusion of multiple
Columnar metaplasia of stratified squamous epithelium activated macrophages to form granuloma.
of the esophagus that increase risk of developing
adenocarcinoma. Group. cross matching (GXM):
To test donor red cells against recipient serum to detect
Bence Jones protein: any potential incompatibility through which antibodies
Monoclonal globulin proteins or IG light chain found in in recipient cause hemolysis to donor cells.
the urine, produced by neoplastic plasma cells.
Present in 2/3 of multiple myeloma cases. Hemorrhoids:
Swollen or inflamed anal cushions.
Breslow thickness:
Measured in mm. From the top of stratum granulosum HER-2:
to the deepest point of tumor involvement. Transmembrane epidermal growth factor and it is
overexpressed in 15% of breast cancer cases and
Carcinoma: associated with bad prognosis.
Is a type of cancer that develops from epithelial cells.
High sensitivity test:
Carcinoid tumor: The ability of a test to correctly identify those with the
Slow-growing type of neuroendocrine tumor originating disease (true positive rate).
in the cells of the neuroendocrine system.
High specificity test:
Cellulitis: The ability of the test to correctly identify those without
Spreading bacterial infection of the skin affects the the disease (true negative rate).
dermis and subcutaneous fat, characterized by redness,
warmth, swelling, and pain. Human Leucocyte Antigens:
Proteins located on surface of WBC.
Choriocarcinoma:
Is a highly malignant form of testicular tumor contain 2 Hyperplasia:
cell types, syncytiotrophoblasts & cytotrophoblasts. Increase in the No. of cells in tissue or organ in
response to a stimulus.
Contact tracing:
The Identification and diagnosis of persons who may Immunohistochemistry
have come into contact with an infected person. Method of localizing specific antigens in tissues or cells
based on antigen antibody recognition.
Cryptorchidism:
Complete or partial failure of the intra-abdominal testes Involucrum:
to descend into the scrotal sac and Is associated with Thickened new bone formed by periosteum enclosing
testicular dysfunction and an increased risk of testicular sequestrum which if infection of the bone persist will
cancer. lead to perforation of involucrum and sinus formation
to the skin through it pus will discharge and causing
DIC: establishment of chronic osteomyelitis.
Pathological consumptive coagulopathy characterized
by widespread hemorrhage, Thrombocytopenia, Malignant melanoma:
decreased fibrinogen, increased FDPs due to activation Malignant neoplasm of melanocytes.
of coagulation and fibrinolytic systems that leads to
formation of microthrombi in many organs with the
consumption of the clotting factors and platelets.
Mohs micrographic surgery:
One of surgical options for treatment of basal cell
carcinoma in which serial tangential horizontal sections

90
90 MO’s MRCS B NOTES (Previously Reda’s called Notes) PATHOLOGY
are taken and examined histologically until all margins
are clear. Pus
Thick, yellowish liquid that is formed as part of an
Metastasis: inflammatory response typically associated with an
Survival and growth of cells at a site distant from their infection and is composed of exudate chiefly containing
primary origin. dead white blood cells (as neutrophils), tissue debris,
and pathogenic microorganisms (as bacteria).
Multiple myeloma:
The most common primary bone tumor in elderly of Sickle cell disease:
plasma cell neoplasm commonly associated with lytic Common hereditary hemoglobinopathy caused by a
bone lesions, hypercalcemia, renal failure & acquired point mutation in 13-globin that promotes
Immune abnormalities. it produces large amounts of polymerization of deoxygenated hemoglobin, that leads
igG 55% or igA 25%. to replacement of glutamate with valine resulting
abnormal hemoglobin (HbS) that responsible for the
Necrosis: disease and causing red cell distortion, hemolytic
Accidental and unregulated form of cell death resulting anemia, microvascular obstruction & ischemic tissue
from damage to cell membranes and loss of ion damage.
homeostasis.
Telomere:
Pathological fracture: Region of repetitive nucleotide sequences at each end
Bone fracture which occurs without adequate trauma of a chromosome, which protects the end of
and is caused by a preexistent pathological bone lesion. chromosome from deterioration or from fusion with
neighboring chromosomes.
Panton Valentine leucocidin (PVL}:
Cytotoxic one of 13-pore-formlng toxins associated with Thrombus:
increased virulence of certain strains of Staphylococcus Solid material formed from the constituents of blood in
aureus. It is present in the majority of community flowing blood.
associated Methicillin resistant Staphylococcus aureus
(MRSA).

91
PATHOLOGY MO’s MRCS B NOTES (Previously called Reda’s Notes) 91
New
Stations
Renal Cell carcinoma (Pathology)

SCENARIO:
Young patient with a background of hyperparathyroidism, has now developed secondary renal failure. He also
has renal stones. Blood results available.

is in
How hyperparathyroidism caused renal failure?
Abnormalities in renal tubular absorption of phosphate → Secondary hyperparathyroidism is
Hyperphosphatemia → acts on parathyroid cells (++ PTH defined as a derangement in Calcium
secretion). hemostasis, which leads to
compensatory increase in PTH secretion.
It occurs primarily as a result of chronic
kidney disease and therefore sometimes
Chronic Kidney
referred to as renal hyperparathyroidism
Disease (↓GFR)

1,25-dihydroxy Phosphate
vitamin D retention

↓ Intestinal High serum


Calcium absorption
phosphorus

Low serum
Calcium

↑ PTH
Synthesis

Secondary
hyperparathyroidism

RCC. Typical cross-section of yellowish, spherical


neoplasm in mid-polar region of kidney. Note

13
the tumor in the dilated thrombosed renal vein

MRCS OSCE B NEW STATIONS 2021


What is the lining epithelium of urinary tract?
Stratified, transitional epithelium

What are the genitourinary complications of stones?


• Deterioration of renal functions
• Obstruction
• Pyelonephritis, pyelonephrosis
• Rupture
• Fistula
• Infection, sepsis
• Carcinoma

Types and Treatment of urinary stones?


Types:
• Calcium oxalate – 85% (second most radio-opaque)
• Cystine – 1% (radio-dense)
• Uric Acid – 5-10% (radio-lucent)
• Calcium Phosphate – 10% (Most radio-opaque) Q11: Management of stones?
According to its size and location in the urinary tract:
• Struvite – 2-20% (mostly associated with infection) - stones less than 5 mm is for conservative management (analgesics, alpha blocker, and
good hydration)
Treatment: - For proximal and mid-ureteral stones that are ≤10 mm, >> SWL or URS (ureteroscopy)
• Less than 5mm – Expectant management - For proximal and mid-ureteral stones that are >10 mm, >> URS
- For all distal ureteral stones, regardless of size, >> URS
• Less than 2cm - Lithotripsy (or URS if pregnant) #For emergency cases: per cutaneous nephrolithotomy (PNL) or URS with ureteral
• Complex renal calculi and staghorn calculi – PCNL stenting.

• Stone any size + obstructed, infected system – Urgent decompression (URS or nephrostomy)

Most common type of Renal Cell Carcinoma?


Clear cell carcinoma is the most common type, accounting for 70-80% of renal cell carcinoma. The tumors are
made up of cells with clear or granular cytoplasm and are non-papillary.

Secon most common renal cell carcinoma?


Papillary carcinoma accounts for 10-15% of renal cancers.
Its is characterized by a papillary growth pattern and occurs in both familial and sporadic form

Risk factors of RCC?


• Tobacco (Smoking)
• Obesity
• Hypertension
• Unopposed estrogen therapy
• Exposure to asbestos, petroleum products and heavy metals (Trichloroethylene)
• ESRD, CKD, acquired cystic disease and tuberous sclerosis

MRCS OSCE B NEW STATIONS 2021


14
Macroscopic and microscopic findings in RCC? (Reference to pathology report) In details

Clear cell cancer (ccRCC) Papillary Carcinoma (pRCC)


Macroscopic • Arises from proximal convoluted tubules • Arises from distal convoluted tubules
Unilateral and Solitary • Bilateral and multiple
Bright yellow-grey-white spherical • Less vibrantly orange yellow because of


masses their lower lipid content
Margins are sharply defined and Papilla formation with fibrovascular cores
confined within the renal capsule



Prominent areas of cystic softening or Necrosis, hemorrhage and cystic
of hemorrhage degeneration



Microscopic • Rounded or polygonal shape and • Cuboidal or low columnar cells arranged
abundant clear or granular cytoplasm, in papillary formations
which contains glycogen and lipid • Interstitial foam cells
• Psammoma bodies may be present
• Stroma is usually empty

What is R1?
• R0 – No residual tumor When dysplasia is severe and involves
• R1 – microscopic residual tumor the full thickness of the epithelium, but
• R2 – macroscopic residual tumor the lesion does not penetrate the
basement membrane, it is referred to as
carcinoma in situ.

How can stones cause cancer?


Stones → Chronic inflammation and infections → altered proliferation in urothelial cells → development of a
tumor

How can irritation lead to neoplasia?


Chronic inflammation → Dysplasia and DNA damage → Cancer

MRCS OSCE B NEW STATIONS 2021


15
Staging?
CT imaging with contrast enhancement of the chest, abdomen and pelvis is required for optimal staging
Staging of RCC based on size, position and lymph node involvement:
• Stage I: Tumor < 7cm is largest dimension, limited to
kidney
• Stage II: Tumor > 7cm in largest dimension, limited to
kidney
• Stage III: Tumor in the major vein or adrenal gland with
intact Gerota’s fascia or regional LN involved
• Stage IV: Tumor beyond gerota’s fascia

MRCS OSCE B NEW STATIONS 2021


16
STATION 3:
Squamous Cell Carcinoma (SCC) On Arm - Pathology
SCENARIO:
Venous ulcer in forearm, not healed, biopsy taken.

PLEASE READ “BCC” SECTION TOO

From biopsy report, what are the criteria of malignancy?


• Invasion: Malignant cells do not respect tissue
boundaries, and can be seen infiltrating or invading
into surrounding structures
• Increased mitotic rate: Malignant cells will often
have increased numbers of mitoses.
• Differentiation and anaplasia: Normal cells are
usually structured in a particular way that
corresponds with their function. This is known as
differentiation. Malignant cells may become less
A
differentiated as part of their path to malignancy.
This is known as anaplasia. Well-differentiated
malignant cells show features similar to the parent
tissue. For example, well differentiated
adenocarcinoma cells will tend to form gland-like
structures; well differentiated squamous cell
carcinomas may show intercellular bridging or
keratin formation. Poorly differentiated cells have
lost most of their resemblance to the parent tissue,
which may be difficult to identify without special
staining techniques. Anaplastic cells have no
resemblance to their parent tissue, and usually B
indicate a very aggressive malignancy.
Invasive SCC
A – A nodule, hyperkeratotic lesion on the ear, associated with
Anaplastic Features: metastasis to a prominent post auricular LN (arrow)
• Loss of normal tissue architecture: Normal B – Tumor invades the dermal soft tissue as irregular
projections of atypical squamous cells exhibiting acantholysis
cells are usually arranged in an orderly
fashion. Epithelial cells often have polarity,
with their nuclei at a specific location. Malignant cells lose this architecture and are arranged
haphazardly
• Pleomorphism: Malignant cells may show a range of shapes and sizes, in contrast to regularly
sized normal cells. The nuclei of malignant cells are often very large (often larger than the
entirety of a normal cell) and may contain prominent nucleoli.
• Hyperchromatic nuclei: The nuclei of malignant cells typically stain a much darker color than
their normal counterparts.
• High nuclear-cytoplasmic ratio: The nuclei of malignant cells often take up a large part of the cell
compared with normal cell nuclei

MRCS OSCE B NEW STATIONS 2021


17
• Giant cells: Some malignant cells may coalesce into so-called giant cells, which might contain the
genetic material of several smaller cells.

Six months later, patent developed cervical lymphadenopathy. How does cancer spread to lymph nodes?
Malignant tumors release growth factors such as VEGF-C to induce lymphatic vessel expansion (lymph
angiogenesis) in primary tumors and in draining sentinel LNs, thereby promoting LN metastasis.
• Permeation
• Embolization

Excision done but not enough, what will you do?


Re-excision With frozen section or Mohs microsurgery

What is frozen section?


It’s a pathological laboratory procedure to perform rapid microscopic analysis of a specimen.

How the specimen is fixed?


The surgical specimen is placed on a metal tissue disc which is then secured in a chuck and frozen rapidly to
about -20 to -30 °C. The specimen is embedded in a gel like medium called OCT and consisting of poly ethylene
glycol and polyvinyl alcohol. Subsequently, it is cut frozen with the microtome portion of the cryostat, the
section is picked up on a glass slide and stained (usually with hematoxylin and eosin, the H&E stain).

What is the advantage of frozen section?


Frozen section technique shortens the time to make a preliminary diagnosis significantly from traditional tissue
biopsy preparation.

What other alternatives to frozen section?


• FNAC
• Imprint Cytology

After excision and grafting, graft became infected and sloughed. Swab shows MRSA. What will you do?
• Inform infection control team/microbiologist
• Follow, infection control protocol
• Wound debridement and regular dressing, If abscess, I&D
• Give antibiotics according to trust/local protocol
Outpatient:
o Oral Antibiotics as clindamycin, amoxicillin plus tetracycline or tmp/smx, linezolid*
Inpatient:
o Vancomycin Dose to target trough level 7-14days
o Linezolid 600 mg twice daily, PO or IV 7-14
o Daptomycin 4 mg/kg once daily 7-14
o Telavancin 10 mg/kg once daily 7-14
o Clindamycin 600 mg IV or 300 mg PO 3times daily
Decolonization with mupirocin nasal or chlorhexidine for body decolonization

MRCS OSCE B NEW STATIONS 2021


18
Define granulation tissue?
Combination of proliferating fibroblast, loose
connective tissue, new blood vessels and scattered
chronic inflammatory cells. Forms a type of tissue
that is unique to healing wounds and is called
granulation tissue.

Fate of granulation tissue?


Remodeling – the connective tissue that has been
deposited by fibroblasts is recognized to produce the
stable fibrous scar. This process begins 2-3 weeks
following injury.

Granulation tissue showing numerous blood vessels,


edema and a loose extracellular matrix containing
associated inflammatory cells. Collagen is stained blue by
the trichrome stain, minimal mature collagen can be seen
at this point

MRCS OSCE B NEW STATIONS 2021


19
STATION 4:
Bleeding Pseudoaneurysm - Pathology
SCENARIO:
Male patient known case of HIV, HBV, HCV presented with red groin mass after injection in groin.

Type of HIV virus? Subtypes of HIV


Single-stranded RNA Retrovirus HIV 1 (more common - 95% of all infection
HIV 2 (less common, less infectious and progresses
more slowly, mainly found in west Africa)
Characteristics of HIV?
Human immunodeficiency virus (HIV) is a retrovirus
transmitted through body fluids
In most cases, HIV is sexually transmitted and occurs by
contact with or transfer of blood, pre-ejaculate, semen and
vaginal fluids

Non-sexual transmission can occur from an infected


mother to her infant during pregnancy, during childbirth by
exposure to her blood or vaginal fluid, and through breast
milk.

Causes of true and false pseudoaneurysm?


True:
• Congenital: Marfan’s, Ehlers-Danlos
• Acquired: atherosclerosis The structure of HIV 1 virion, the viral particle is covered by
• Infection: Syphilis a lipid bilayer derived from the host cell and studded with
viral glycoproteins gp 41 and gp 120
False:
• Trauma: IV Injections, horse rides “popliteal
aneurysm”
• Surgery

Which layer is affected in pseudoaneurysm?

MRCS OSCE B NEW STATIONS 2021


24
Sub-intimal layer

Why does he have bleeding?


• From the trauma
• Liver involvement (HCV and HBV) affecting coagulation

You keep pressure on wound, but bleeding has not stopped, why?
Because there is liver involvement and affection of VIT K dependent clotting factors 2,7,9,10 and
thrombocytopenia

Precaution when stop bleeding?


Apron, eye protection, double gloves,
cap, mask

Sequence of clotting?
Injury to blood vessels → Platelets
aggregation → Platelet plug →
activation of intrinsic pathway

Why are clotting factors not working


properly?
• Due to liver affection where Injury to a blood vessel exposes collagen and thromboplastin, recruiting platelets to the
most of the factors are site of injury to form a temporary plug. Platelets release 5-hydroxytryptamine, among
manufactured (2,7,9,10 – VIT other factors, resulting in smooth muscle contraction and vasoconstriction. Activation of
the clotting cascade in response to collagen and thromboplastin activates thrombin,
K dependent) which converts circulating fibrinogen to fibrin monomers. Fibrin monomers polymerize
• Lower CD4 count, damage to and are cross-linked and accumulate with platelets at the site of injury to form the
the bone marrow where blood definitive clot.
cells are produced.

MRCS OSCE B NEW STATIONS 2021


25
STATION 2:
Gastrointestinal Stromal Tumor (GIST)
STATION 7:

You might also like